You are on page 1of 72

EVIDENCE Prof.

OShea

Introduction Evidence law is about the limits we place on the information juries hear. Origin (including 2011 restyling): o Codified in about 1975 by statute; prior to that, there were no written rules of evidence. o Rules Advisory Committee -> SCOTUS -> Congress RAC meets to see about changes that need to be made. Supreme Court then meets to see what they think should happen and make the specific rules Goes to Congress. Proposed rules must be given to Congress by May 1. If Congress does not act to the contrary, the rule is automatically put into place Dec. 1. Congress can change the rules because they are essentially statutes. Legislative history is important: The language of a rule itself should be taken as the prime source of meaning, the rule should be read in the light of its legislative history, which on occasion may even override an apparently plain and unmistakable meaning of the words of the rule. P. 4 When a rules language and history both fail to make its meaning clear, look to the common law for guidance.

CHAPTER 1 GENERAL PRINCIPLES OF RELEVANCE Probativeness and Materiality (pp. 18-29) Rule 401. Definition of Relevant Evidence Relevant evidence means evidence having any tendency to make the existence of any fact that is of consequence to the determination of the action more probable or less probable than it would be without the evidence. o Lenient test evidence need not prove anything conclusively; it merely must have some tendency to make a fact more or less probable. o Evidence will be probative if it contributes just one brick to the wall of proof built by a party. o Liberal thrust of the rules of evidence a preference for more rather than less evidence. Evidence must be material it must bear on a fact that is of consequence to the determination of the action. o Whether evidence is material turns on what issues are at stake in the proceeding which often turns on the substantive law of the jurisdiction. To decide whether an issue is material, dont look to the evidence rules for an answer, but to the substantive law. Evidence must be probative of a material fact. o The evidence must have a tendency to make the existence of that fact more probable or less probable than it would be without the evidence. o At common law, probativeness was also known as relevance (confusing when considering the wording of Rule 401)

Some authorities use the expression logical relevance

Rule 402. Relevant Evidence Generally Admissible; Irrelevant Evidence Inadmissible All relevant evidence is admissible, except as otherwise provided by the Constitution of the United States, by Act of Congress, by these rules, or by other rules prescribed by the Supreme Court pursuant to statutory authority. o Evidence which is not relevant is not admissible. o Rule 401 is a very low threshold (any tendency) o Rule 402 says to look to the other rules, statutes, etc. to determine if evidence is admissible. Rule 403. Exclusion of Relevant Evidence on Grounds of Prejudice, Confusion, or Waste of Time Although relevant, evidence may be excluded if its probative value is substantially outweighed by the danger of unfair prejudice, confusion of the issues, or misleading the jury, or by considerations of undue delay, waste of time, or needless presentation of cumulative evidence. Rule 1101. Applicability of Rules (a) Courts and Judges. a. These rules apply to the United States district courts, the District court of Guam, the District Court of the Virgin Islands, the District Court for the Northern Mariana Islands, the United States courts of appeals, the United States Claims Court, and to the United States bankruptcy judges and United States magistrate judges, in the actions, cases, and proceedings and to the extent hereinafter set forth. The terms judge and court in these rules include United States bankruptcy judges and United States magistrate judges. (b) Probativeness (pp. 20 29) Relevancy exists as a relation between an item of evidence and a proposition sought to be proved. If an item of evidence tends to prove or disprove any proposition, it is relevant to that proposition. An offered item of evidence may be excluded as irrelevant because it is not probative of the proposition at which it is directed, or because that proposition is not provable in the case. Brief Overview of Trial Structure A complaint is filed. If its a civil action, the action is brought by a private party (plaintiff). If its a criminal action, it is brought by the state or federal government. Starts with Ps case in chief. o P calls its first witness (W1). o P presents Ws testimony (direct examination) nonleading questions. o D cross examines witness can use leading questions Wants to counteract/weaken Ws testimony and/or her effect on the jury. Leading questions: Youd been drinking, hadnt you? etc. o P can then redirect and so on.

P can call another witness (could include an expert witness) Witness testimony is only one form of evidence at trial. Plaintiff can also introduce real evidence (e.g., brass knuckles in an assault and battery case), documentary evidence (e.g., medical records, prior convictions, etc.) Plaintiff rests D can move for a directed verdict saying that P did not present enough evidence to prove its case. If not. . . D calls its W1. o Direct examination o Cross examination o Redirect o Etc. Jury deliberates

o o

Problem 1.1 Show me the body! (probativeness requirement) Defense objected to wifes statements to the police Where is the body? Show me the body! as irrelevant. An offered item of evidence may be excluded as irrelevant because it is not probative of the proposition at which it is directed. The husband was arrested for murder. The evidence in dispute is the wifes statements. Defense objects on the grounds of relevance. (If the evidence is deemed irrelevant, it would be because it failed the probativeness test) Prosecutors want to use her statements to infer that she had some information (that there was no body) Rule 401 states that evidence is relevant if it has any tendency to make a fact more or less probable than it would be without the evidence (remember: low threshold) . Problem 1.4 Knowledge (materiality requirement) Defendant testified that she did not know until her arrest in this case that assault and battery is punishable in Massachusetts by more than a years confinement. The prosecution objected to the testimony as irrelevant. How should the court rule? To be material, evidence must bear on a fact that is of consequence to the determination of the action. Here, the statute states that it is unlawful for a person who has been convicted in any court of a crime punishable by imprisonment for a term exceeding one year to ship or transport in interstate or foreign commerce, or possess in or affecting commerce, any firearm or ammunition. Since Defendant could have been sentenced to 2 years in prison, the statute applies to her.

Problem 1.7

The statute does not set the mental state, so reckless or negligence would satisfy the culpability level. As a result, whether Defendant knew she could have been sentenced to more than a year is irrelevant and the prosecutions objection should be sustained. If evidence is relevant, is it admissible? Relevance is the most basic threshold. Its the first question we ask about evidence. United States v. James o Summary: James handed her daughter, Jaylene, a gun that was subsequently used to kill Ogden. James claims she gave her daughter the gun because was fearful for her daughter. o Evidence in dispute: the police records of Ogdens prior convictions. o James lawyer: use the evidence to support James contention that Ogden was a violent man. It would give credibility to the defense of self defense. It would show that James was not paranoid and had reason to fear. o Prosecution: James did not have Ogdens records. o The ninth circuit ruled in favor of letting the evidence in.

Unfair Prejudice (Waste of Time, etc.) FRE 403 Photos and Other Inflammatory Evidence State v. Bocharski, 22 P.3d 43 (Ariz. 2001). Summary: Bocharski was convicted of murder and burglary. He appealed his conviction, arguing that the trial court erroneously admitted into evidence photographs of the victim that were "gruesome, highly inflammatory, and unduly prejudicial." Court decision: admitting two of the photos was incorrect, but the error did not contribute to or affect the jurys verdict (harmless error, not reversible) The defendant did not contest the fact that is of consequence (the victim died as a result of being stabbed) The prosecution didnt make use of them no testimony, did not show defendants knife in the wound Pictures had done some work on her the gruesomeness of the photos of her skull with the rod was particularly gruesome because of what the prosecution had done (inserted a metal rod in to the skull) The defendants knife was not in the wound in the photo. o Note: If it were, the probative value would have dramatically increased, and it would probably have been enough to change the result of the ruling. Probative value would not have been significantly outweighed by the unfair prejudice Problem 1.8 Summary: A man is on trial for possessing an unregistered machine gun. The government alleged he had altered a semiautomatic rifle so it would discharge more than one shot per trigger pull. Evidence in dispute: a photo of the gun surrounded by other weapons

Argument for allowing the evidence: it shows the gun is clean counteracting the defense argument that it was dirty and misfired. Argument against allowing the evidence: it is unfairly prejudicial because it does not show the condition of the gun; the point of the photo was to show he had many weapons (and hes a scary person who deserves punishment) Decision: Relevance some commentators state that the gun is not relevant because it doesnt show the inside of the gun. Assuming it is relevant, though, it should be excluded because the probative value is substantially outweighed by the unfair prejudice. o Low probative value the stated purpose of the photo was to show it was clean, but the photo only showed the outside of the gun. It doesnt necessarily prove the stated goal. o It would prejudice the jury to the effect that it would persuade the jury that he should be locked up on principle, not because he is guilty o Misleading the jury He only owned the gun the other weapons were his roommate. Commonwealth v. Serge, 586 Pa. 671, cert. denied, 127 S.Ct. 275 (2006). Summary: Michael Serge, a former Lieutenant of detectives with the Scranton Police Department, was convicted of murdering his wife, Jennifer. Michael said he killed her in self defense. Evidence in dispute: Commonwealth introduced a computer generated animation (CGA) depicting the scene of expert testimony. Argument for allowing the evidence: it depicted the event so that the jury could see and understand. Argument against allowing the evidence: Defense objected on the basis that it was prejudicial (visual representation of the scene is inherently prejudicial because the visual appeal to a jury could result in an acceptance of the CGA as fact and the cost was prohibitive for defense to produce its own). Rule: A CGA should be deemed admissible as demonstrative evidence if it: (1) is . . . a fair and accurate representation of the evidence it purports to portray; (2) is relevant pursuant to Pa. R. Evid. 401 and 402; and (3) has a probative value that is not outweighed by the danger of unfair prejudice pursuant to Pa. R. Evid. 403 Courts decision: The CGA was not prejudicial. Notes:

United States v. James, 169 F.3d 1210 (9th Cir. 1999) en banc (dissent in James case above) Summary: James is on trial for giving a gun to her daughter, Jaylene, who killed James boyfriend, Ogden. Evidence in dispute: Ogdens criminal record Argument for allowing the evidence: would show that James had a reason to fear her boyfriend Argument against allowing the evidence: courts should only allow in the evidence that was known to James at the time; allowing in the evidence could facilitate the attitude he was a bad man and got what he deserved.

Court decision: The court admitted the evidence because it supported the contention that she had reason to fear. Evidence of Flight United States v. Myers, 550 F.2d 1036 (5th Cir. 1977), cert denied, 439 U.S. 847 (1978). Summary: Myers was arrested for robbing a bank in Florida Evidence in dispute: flight he left Orlando 3 6 weeks after the robbery then might have attempted to flee after being Argument for allowing the evidence: shows a consciousness of guilt Argument against allowing the evidence: Its actually inconclusive that he was fleeing. He was chased by agents in plain clothes in Florida, nearly run over by an unmarked car with an officer out of uniform in California, and may or may not have moved from his motorcycle in California (testimony changed). Also, he had committed a robbery in Pennsylvania, too. If he was fleeing in California, he could have been fleeing from the guilt of that robbery. Inconsistent evidence of flight in California o In trial for Pennsylvania robbery, officer said it didnt appear that anyone was fleeing. o In trial for Florida robbery, the officer then changed his story. Court decision: evidence inadmissible; the judgment and conviction appealed from must be reversed. Notes: Four inferences must hold true to allow flight as evidence: 1. From the defendants behavior to flight; 2. From flight to consciousness of guilt a. defendant fled because he knew he was guilty of committing an illegal act 3. From consciousness of guilt to consciousness of guilt concerning the crime charged; a. suspect knows hes guilty of robbing a store, but hes being chased for murder; not consciousness of guilt concerning the murder, but conscious of guilt for robbery 4. From consciousness of guilt concerning the crime charged to actual guilt of the crime charged. a. instead of I was afraid they were going to pin something on me or frame me for something (Interesting note: Officer Hanlon was not allowed to testify at the new trial.) So, Defendants behavior is actually flight because he was conscious of his guilt of the crime charged and he was actually guilty of that crime (not simply afraid). Fuhrman tapes (from the O.J. trial) -> probabtive value was impeachment of witness. STIPULATIONS Def: a voluntary agreement between opposing parties concerning some relevant point; esp., an agreement relating to a proceeding made by attorneys representing adverse parties to the proceeding.

Old Chief v. United States, 519 U.S. 172 (1997). Summary: Old Chief was arrested and tried for possession of a firearm in violation of 18 U.S.C. 922(g)(1) which prohibits possession of a firearm by anyone with a prior felony conviction, which the government can prove by introducing a record of judgment or similar evidence identifying the previous offense Issue: Whether a district court abuses its discretion if it spurns the offer [of a stipulation] and admits the full record of a prior judgment, when the name or nature of the prior offense raises the risk of a verdict tainted by improper considerations, and when the purpose of the evidence is solely to prove the element of prior conviction. Holding: it does. Evidence in dispute: Old Chiefs prior record assault causing serious bodily injury Argument for: 1) establishes a necessary element of a prior conviction; 2) the prosecution is to be allowed to try the case in the manner it sees fit. Argument against: risk of unfair prejudice It unfairly prejudicial to the defendant because jury might see the name and nature of the prior conviction and convict on this count based on the prior instead of on the merits of this particular case. (Hes a bad guy and should be punished) - Such improper grounds certainly include the one that Old Chief points to here: generalizing a defendants earlier bad act into bad character and taking that as raising the odds that he did the later bad act now charged (or, worse, as calling for preventive conviction even if he should happen to be innocent momentarily). Is it relevant? Yes; the government can introduce a defendants prior record to show the statute applies to him. It proves the matter asserted. Is it admissible under 402? Is it admissible under 403? Notes:

CHAPTER 2: SPECIALIZED RELEVANCE RULES (UNFAIR PREJUDICE RULES) FRE 407 411 (pp. 90 134) Rules 407, 408, 409, and 411 have a different structure. Basically, they say this evidence is not admissible for this purpose in this type of case. But for any other purpose it can be offered and is not barred for that rule.

Rule 407. Subsequent remedial measures (a) Not admissible to prove negligence, culpable conduct, product defect, need for a warning label. (b) The court may admit this evidence for another purpose, such as impeachment or if disputed proving ownership, control, or the feasibility of precautionary measures. Rule 408. Compromise and offer to compromise (a) Prohibited Uses. The following is not admissible when offered to prove liability for, invalidity of, or amount of a claim that was disputed as to validity or amount, or to impeach through a prior inconsistent statement or contradiction: (1) Furnishing, promising, or offering or accepting, promising to accept, or offering to accept a valuable consideration in compromising or attempting to compromise the claim; and (2) conduct or a statement made during compromise negotiations about the claim except when offered in a criminal case and when the negotiations related to a claim by a public office in the exercise of regulatory, investigative, or enforcement authority. (b) Exceptions. The court may admit this evidence for another purpose, such as proving a witnesss bias or prejudice, negating a contention of undue delay, or proving an effort to obstruct a criminal investigation or prosecution. Rule 409. Offers to pay medical and similar expenses (a) Evidence of furnishing, promising to pay, or offering to pay medical, hospital, or similar expenses resulting from an injury is not admissible to prove liability for the injury. Rule 411. Liability Insurance (a) Evidence that a person was or was not insured against liability is not admissible to prove whether the person acted negligently or otherwise wrongfully. But the court may admit this evidence for another purpose, such as proving, a witnesss bias or prejudice or proving agency, ownership, or control. Rule 410. Pleas, Plea Discussions, and Related Statements (a) Prohibited Uses. In a civil or criminal case, evidence of the following is not admissible against the defendant who made the plea or participated in the plea discussions: (1) A guilty plea that was later withdrawn; (2) A nolo contendere plea; (3) A statement made during a proceeding on either of those pleas under Federal Rule of Criminal Procedure 11 or a comparable state procedure; or (4) A statement made during plea discussions with an attorney for the prosecuting authority if the discussions did not result in a guilty plea or they resulted in a later-withdrawn guilty plea. (b) Exceptions. The court may admit a statement described in Rule 410(a)(3) or (4):

(1) In any proceeding in which another statement made during the same plea or plea discussions has been introduced, if in fairness the statements ought to be considered together; or (2) In a criminal proceeding for perjury or false statement, if the defendant made the statement under oath, on the record, and with counsel present. Subsequent Remedial Measures (pp. 94 107) Problem 2.1 Wolf attack Summary: Kenneth Poos had been keeping a wolf at his home. The wolf attacked a beagle, so Kenneth put the wolf on a 6 foot chain. The next day, a neighbor sent her three-year-old to get his older brother. The child had to go past the wolf. The wolf attacked the child. Assume that both the beagles owner and Daniel (through his parents) sue Kenneth Poos for damages caused by the wolf. And assume that each plaintiff, at separate trials, attempts to introduce evidence that Poos chained the wolf after learning of Sophies attack on the beagle. If Poos objects to admission of the evidence at both trials, how should each court rule? What policy benefits might justify the answers Rule 407 gives to this question? Beagle case: Rule 407 prohibits the introduction of subsequent measures to prove negligence, so the beagles owner could not submit into evidence the fact that Poos chained the wolf after the attack to prove negligence. The courts do not want to punish a person for getting wiser thereby discouraging him or her from taking precautions to avoid further problems. If the beagles owner wanted to bring in the evidence for another reason, would have to pass a 403 balancing test. Toddler case: Chaining the wolf is not a subsequent measure, so its not subject to exclusion under Rule 407. Tuer v. McDonald, 347 Md. 507 (1997) Summary Plaintiff's surgery was delayed. The hospital discontinued the use of Heparin. The Plaintiff had a heart attack and died. Subsequently, the hospital changed their policy regarding discontinuing the use of Heparin prior to surgery. The plaintiffs try to make use of the feasibility aspect of Fed R. Evid. 407, which establishes that you can circumvent the subsequent remedial measures rule, if it is used to controvert feasibility. Rule of Law and Holding The feasibility exception of Fed R. Evid. 407 establishes that you can circumvent the subsequent remedial measures rule, if it is used to controvert feasibility. If the defense had just continued to say that they made the best use of the information they had at the time rather than saying it would have been dangerous, they would not have fallen into the feasibility trap of Fed. R. Evid. 407. Notes on Rule 408 (pp. 111 114)

Claim: Rule 408 does not protect offers to compromise made before a claim of some sort has been made. o A lawsuit is a claim, and sometimes a court will deem informal oral or written demands to be claims. o The implicit or potential claim does not mean that all offers of aid or compensation are necessarily covered by FRE 408. Disputed as to Validity or Amount: the policy considerations which underlie Rule 408 do not come into play when the effort is to induce a creditor to settle an admittedly due amount for a lesser sum. Hence the rule requires that the claim be disputed as to either validity or amount. Operation in Criminal Cases: o Rule 408 addresses the admission at criminal trials of conduct and statements made in civil compromise talks. o Except in one fairly narrow circumstance, statements made during compromise negotiations of disputed civil claims are not admissible in subsequent criminal litigation, when offered to prove liability for, invalidity of, or amount of those claims. Allowing them into evidence could lead to parties refusing to admit fault, even if by doing so they could favorably settle the private matter. o The narrow exception: admission at criminal trials of conduct or statements made in civil compromise negotiations when those negotiations related to a claim by a public office or agency in the exercise of regulatory, investigative, or enforcement authority. Where an individual makes a statement in the presence of government agents, its subsequent admission in a criminal case should not be unexpected. May evidence of compromise and of statements be used to impeach? o No Rule 408(a) bars evidence of compromise attempts and of conduct or statements made in settlement talks when offered to . . . impeach through a prior inconsistent statement or contradiction. Cant use the fact that they were willing to settle instead of going to court to imply wrong-doing to fault. o Yes Rule 408(b) permits evidence of compromise negotiations and of conduct and statements made in such negotiations when offered to prove a witnesss bias or prejudice. Evidence of a witnesss interest in the outcome of a case tends to show a motive to lie and hence that the witness is lying. Example: A car accident injured two persons, A and B, both of whom sue the defendant. If the defendant settles generously with A, and A later testifies against B on the defendants behalf, B could offer evidence of As handsome settlement to show her bias toward the defendant.

States Encourage Docs to Apologize by Ray Henry, Associated Press, Wash. Post, April 11, 2007 This is an article about the Im sorry laws passed in several states. It is part of a movement in the medical industry to encourage doctors to promptly and fully inform patients of errors and, when warranted, to apologize.

Favorite quote of the article: Apology is not about confession.

Comparing Rules 408 and 409 (pp. 114 -116) Conduct and statements in compromise negotiations Rule 408 applies to statements made in compromise negotiations, not excited utterances Rule 409 applies to offers to pay medical costs, but not to any statement surrounding the offer o For example: Im sorry I ran the red light! Let me take care of your treatment! Rule 409 bars evidence of the drivers humane impulse to pay the others medical bills The apology and admission of fault will be admissible. The Relevance-Based Rationale o the statement Let me take care of your treatment may be entirely the product of humane impulses and therefore not probative of negligence. o The statement Im sorry I ran the red light let me take care of your treatment sounds like a genuine admission of fault that may be highly probative of negligence. The Public Policy Rationale o Rule 408 aims in part to encourage settlement o Rule 409 aims in part to encourage offers to assist (which may avoid a later lawsuit) Example: Three statements uttered after an accident 1) Oh my goodness, let me help you get that arm treated. Not barred by Rule 408 there are no settlement negotiations going on i. If you can make the case that it is understood that once an accident happens, it follows that there is a disputed claim. Excluded under Rule 409 offer to pay medical expenses cannot be used as evidence to prove negligence or fault i. If this is just an offer to call 911, then it is not excluded; its just an offer to render aid, not pay for it. 2) Suppose I cut you a check for $1,000 for your fender and we call it even. Not excluded under Rule 408 there is no disputed claim i. But you could make the argument that it is understood that once an accident happens, it follows that there is a disputed claim. ii. The speaker acknowledges that there will be a dispute and hes just beginning the negotiations. 3) Man, I knew I shouldnt have smoked that third joint this morning. Not excluded under Rules 408 or 409 admission of liability But 408(a)(2) conduct or a statement made during compromise negotiations about the claim are inadmissible to prove the validity of a disputed claim, so if you can show that settlement negotiations had begun, it can be excluded.

Compromises with Third Parties Neither Rule 408 nor 409 limits its exclusionary reach to compromises or payments between the two parties in this lawsuit. These rules also bar evidence that one of the parties in the suit settled with a third party if that evidence is offered to prove liability for or invalidity of the claim. Such evidence may be admissible, however, to show a witnesss bias. Liability Insurance FRE 411 (pp. 116 122) Two rationales for the rule: 1) Its not probative of liability or negligence if the person carried insurance 2) Public policy we want people to be insured Williams v. McCoy, 550 S.E.2d 796 (N.C. App. 2001) Summary: Joanne C. Williams (Plaintiff) appealed from a judgment entered pursuant to a jurys verdict finding Mia McCoy (Defendant) negligent and awarding Plaintiff $3000 in damages. Williams was not allowed to testify concerning insurance. Plaintiff appealed on the grounds that not allowing her to testify about the insurance was prejudicial to her. It made her look litigious and greedy. o Plaintiff said her explanation was admissible for a purpose other than to prove the existence of liability insurance and that the court abused its discretion in not admitting it as such. She wanted to show that she hired an attorney after a negative experience with Defendants insurance adjuster. She thought that Defendants insurance would pay for her injuries, but after meeting the adjuster, she realized that she was mistaken. Disputed evidence: Defendant was insured. Holding: the trial court abused its discretion in failing to admit plaintiffs explanatory testimony, and the courts abuse of discretion constituted reversible error. Judgment was reversed and the case was remanded for a new trial. o Rule 411 did not bar plaintiffs explanation as to why she hired an attorney. . . Defense counsel knew plaintiff would testify that her motivation for hiring an attorney was a negative encounter with defendants insurance adjuster. o Must do a 403 analysis Explaining why she hired an attorney may have had some prejudicial effect on defendant; however, the prejudice did not outweigh the probative value of plaintiffs testimony and the prejudice she suffered in not being allowed to explain her answer. Regarding prejudice to defendant, most jurors drive automobiles and would more than likely know that in all probability there is insurance, that the matter has been investigated by the insurers claim agent or attorney, and that insurer has employed the trial counsel. Plus, the court could have given a limiting instruction (Rule 105) to limit any prejudice.

Problem 2.5 Claims Adjuster Summary: Joan Lacher attended a bridge club gathering at the Andersons home. While looking for a bathroom, Joan opened a door, stepped into the darkness, and fell down a flight of stairs. While in the hospital, Joan voluntarily gave a tape-recorded statement describing her recollection of the accident to Bob Smith, an adjuster for the Andersons insurance company. The trial court did not allow evidence that Mr. Smith was employed by the insurance company. The Andersons used the tape-recorded statement to impeach Joan at trial. Mr. Smith did not testify. The Lachers argued that evidence of Smiths employment was relevant to show his bias and prejudice in taking Joans statement. At trial, Joan asserted she was mistaken in some particulars when she made the statement to Smith in the hospital. Was the trial judge right to exclude evidence of Smiths employment? What are the best arguments on both sides? Rule: evidence that a person was or was not insured against liability is not admissible to prove whether the person acted negligently or otherwise wrongfully. But the court may admit this evidence for another purpose, such as proving a witnesss bias or prejudice or proving agency, ownership, or control. Analysis: The trial judge was right to exclude the evidence under Rule 411. It is permissible to introduce evidence of liability insurance to show witness bias, but the probative value is substantially outweighed by the risk of unfair prejudice. Mr. Smith did not testify at trial. There is very little probative value in Mr. Smiths identity, but there is substantial risk of prejudice to the Andersons. Andersons argument: Rule 411 prohibits evidence that a person was or was not insured against liability to prove whether the person acted negligently. This is a negligence suit, so the evidence is improper. Who recorded the tape is not relevant. The tape simply gave Joans side of the story immediately after the accident. Lachers arguement: The Lachers want to use Mr. Smiths employment to prove a witnesss bias. The Lachers are not asserting that the fact that the Andersons had insurance implied liability. Instead, they are showing that the tape recording was not done by a disinterested third party. Bob Smith was an employee/agent of the insurance company. He had a personal stake as an agent of the company in the outcome of the trial. Pleas in Criminal Cases FRE 410 (pp. 127 133) Problem 2.8 Prosecutors Offer Summary: A man is accused of first degree murder. Before trial, the prosecutor offered to permit the defendant to plead guilty to second-degree murder. Defendant refused the offer. At trial, Defendant wants to introduce the offer into evidence to show 1) the weakness of the governments case; and 2) the Defendants display of consciousness of innocence in rejecting the offer (a guilty person would have taken it). Should the trial judge permit the offer? Rule: evidence of the following is not admissible against the defendant. . . Analysis: Strictly speaking, the offer should be admitted. The rule does not bar defendant from presenting the evidence. It bars the plaintiff. The defense is the party attempting to put the plea offer

into evidence. However, some courts will ignore the strict language and will not allow the evidence at all. United States v. Biaggi, 909 F.2d 662 (2d Cir. 1990). Summary: Mariotti was offered immunity by prosecutors in exchange for his testimony against other participants in his crime. Mariotti rejected the immunity offer and claimed he was innocent. Mariotti claimed it was error to reject evidence that the prosecutor offered him immunity because it was probative of his consciousness of innocence when he rejected it. Holding: the exclusion of evidence prevented defendant from receiving a fair trial. Rule: Evidence of defendants innocent state of mind is not excluded even when this evidence is obtained during negotiations. Notes: Notes on Rule 410: Pleas, Plea Discussions, and Related Statements Breadth of Exclusion The evidence Rule 410 addresses is always barred except where specifically permitted as opposed to the other specialized relevance rules that exclude evidence only when offered to prove certain specified things, while admitting it on any other issue. May Evidence of Statements be Used to Impeach? Statements the defendant makes during plea negotiations with the prosecutor may not be used to impeach her should she later testify differently at trial. Rationale: if defendants worried that any statements they make during plea negotiations might be used to impeach them at trial, they might not enter plea negotiations. Rule 410 does permit certain statements by defendants to be used against them at later perjury prosecutions an exception that may discourage defendants from being completely candid and open during plea negotiations. Prosecutors may demand that defendants agree that any statements they make during negotiations may be used to impeach any contradictory testimony they give at trial. o Rationale: A criminal defendant may knowingly and voluntarily waive many of the most fundamental protections afforded by the Constitution and there is no reason to assume Congress intended the protection of Rule 410 to be nonwaivable. Explaining Rule 410s Absolute Protection One explanation for the broad protection against evidence of plea negotiations and related statements is the justice systems strong desire to encourage plea bargaining. o Increase efficiency of the justice system dont have to bog down the system prosecuting as many people Also, many criminal defendants might simply refuse to engage in plea negotiations were it not for the broad protection of Rule 410.

Plea Negotiations with the Prosecutor The protection offered by the fourth paragraph of Rule 410 reaches only statement made in the course of plea discussions. If the defendant unilaterally offers information without first establishing that he is seeking a concession, a court may determine that no plea discussions had begun and that the defendants statements are therefore admissible against him. The fourth paragraph is also limited to discussions with prosecutors. o If the prosecutor has designated a police officer to act as an agent for the purpose of plea discussions, statements made to the police officer will fall within the rules protection, but o Defendants speak at their peril to police officers who merely appear to have authority to negotiate pleas. But, some courts will exclude defendants statement if she exhibited an actual subjective expectation to negotiate a plea and that expectation was reasonable given the totality of the objective circumstances. Evidence Offered Against the Prosecutor Rule 410 bars evidence of pleas and plea discussions only when offered against the defendant. But since admitting such evidence would discourage prosecutors from negotiating pleas and would thereby frustrate the purpose of the rule, some courts have ignored the strict language of the rule and have barred the evidence. The Specialized Relevance Rules: Afterthoughts The specialized relevance rules are concrete applications evolved for particular situations of Rule 403. If there were no specialized rules: o Replacing specific exclusionary rules with multiple ad hoc decisions of the trial judge would vastly expand judicial discretion. Could breed arbitrary and biased decision making Could spur forum shopping, as litigants seek out judges whose evidence rulings most nearly suit their needs. o The loss of specific exclusionary rules would spell a loss of predictability Unable to foresee which evidence would come in and which stay out, litigants could not reliably predict trial outcomes and therefore could not reach informed settlements ahead of trial. A loss of predictability would frustrate the substantive policies behind Rules 407 0 410, designed to promote repairs and settlement. o Trial preparation would grow more complex as litigants tried to anticipate all possible outcomes of the judges discretionary weighing tests o Court business would slow as the already over-complicated congeries of pretrial procedures were supplemented by long lists of in limine motions brought to secure evidence rulings in advance of trial.

Because criminal defendants have rights that might be left unprotected by ad hoc rulemaking, there probably would have to be different rules for criminal and civil litigation.

CHAPTER 3: CHARACTER EVIDENCE Class Notes: Bennett Marco: Raymond Shaw is the kindest, braves, warmest, most wonderful human being Ive ever known in my life. Direct character evidence Rule 404 imposes strict restrictions on admissibility Indirect character evidence (a.k.a., propensity evidence) Says that this person has a tendency to act a certain way. It is possible that in this case, the defendant acted the same way; therefore, he should be found guilty. Old Chief case Old Chief did not want the evidence in because the inference that he committed the crime would convict him for the current one regardless if the state had proven its case. Character as Evidence of Conduct The propensity argument the justification of using proof of character as substantive evidence of conduct on a particular occasion. This is ALWAYS inadmissible!!!!!! *****Buzz words: General prohibition against the propensity inference***** Rule 404 Character Evidence; Crimes or Other Acts (a) Character Evidence (1) Prohibited Uses. Evidence of a persons character of character trait is not admissible to prove that on a particular occasion the person acted in accordance with the character or trait. (2) Exceptions for a Defendant or Victim in a Criminal Case (A) A defendant may offer evidence of the defendants pertinent trait, and if the evidence is admitted, the prosecutor may offer evidence to rebut it; i. In other words, the defendant must open the door to enter character as evidence (B) Subject to the limitations in Rule 412, a defendant may offer evidence of an alleged victims pertinent trait, and if the evidence is admitted, the prosecutor may: i. Offer evidence to rebut it; and ii. Offer evidence of the defendants same trait; and (C) In a homicide case, the prosecutor may offer evidence of the alleged victims trait of peacefulness to rebut evidence that the victim was the first aggressor. (3) Exceptions for a Witness. Evidence of a witnesss character may be admitted under Rules 607, 608, and 609 (Those are the rules for impeachment; not just for a criminal case, can impeach witnesses in civil cases, as well.)

The Exceptions to the Propensity Inference in FRE 404(a) (b) Crimes, Wrongs, or Other Acts. (1) Prohibited Uses. Evidence of a crime, wrong, or other act is not admissible to prove a persons character in order to show that on a particular occasion the person acted in accordance with the character. (2) Permitted Uses; Notice in a Criminal Case. This evidence may be admissible for another purpose, such as proving motive, opportunity, intent, preparation, plan, knowledge, identity, absence of mistake, or lack of accident. On request by a defendant in a criminal case, the prosecutor must: (A) Provide reasonable notice of the general nature of any such evidence that the prosecutor intends to offer at trial; and (B) Do so before trial or during trial if the court, for good cause, excuses lack of pretrial notice. People v. Zackowitz (N.Y. 1930) Summary: Defendant Zackowitz shot and killed Coppola. Defendant claims self defense; the prosecution argues premeditation. The question was the degree of the killing, not whether Defendant committed the murder. Evidence in dispute: three pistols and a tear-gas gun found in a radio box at Zackowitzs home. (Ha ha! He said, In Oklahoma, thats just a good start!) o Defendant: the admission of the guns kept him from getting a fair trial because the jury had decided that he was a violent man and must have premeditated the shooting of Coppola. o Prosecution: the purpose of laying the weapons before the jury was to bring persuasion that here was a man of vicious and dangerous propensities, who because of those propensities was more likely to kill with deliberate and premeditated design than a man of irreproachable life and amiable manners. Rule: character is NEVER an issue in a criminal prosecution unless the defendant chooses to make it one. o The Defendant must open the door. Notes: There could be scenarios where the evidence would be admissible. None were offered here. Prosecution admits that it was offered as propensity evidence. The Propensity Box Rule: Evidence that a person has a particular character trait generally is not admissible to show that the person acted in conformity with that trait at a particular time. The evidence is not not relevant (so it is relevant). The problem is that such evidence can cause unfair prejudice. Two forms of unfair prejudice: o First: the jury might give excessive weight to the vicious record of crime thus exhibited, and allow it to bear too strongly on the present charge. o Second: The jury might take proof of character as justifying a condemnation irrespective of guilt of the present charge the jury might punish a person simply for being a man of bad character, on the theory that this sort of person is better kept off the streets even if he is not guilty of the crime charged or

The jury might punish a person for the crime entered into evidence deserves punishment even if he is not guilty of the crime charged Additionally, the specific acts used to prove character normally are not the focus of the case. Evidence about them could confuse and distract the jury. And once litigant offers evidence that someone committed certain specific acts, the other side will want to offer evidence that she didnt. The mini-trial that ensues could consume gobs of time. (Rule 403 balancing)

Diagramming the Propensity Box This is the path that the prosecution was trying to take the jury.
Zachowitz owns lots of guns. Zachowitz is a dangerous and vicious character. Zachowitz is likely to act in conformity (this step in the chain is barred) Zachowitz committed the crime with premeditation.

The problem was that the path the jury had to go down was an impermissible path. Routes Around the Propensity Box and Close up of Rule 404(b) Evidence may be admissible to show proof of motive, opportunity, intent, preparation, plan, knowledge, identity, or absence of mistake or accident. To prove identity or that he was at the crime scene this is his gun, it was at the crime scene, so he must have been at the crime scene. The judge would give a Rule 105 limiting instruction to the jury: o When evidence which is admissible. . . for one purpose but not . . . for another purpose is admitted, the court, upon request, shall restrict the evidence to its proper scope and instruct the jury accordingly. o The judges instruction should explain in clear terms the permitted, around-the-box purpose for which the evidence is admitted. If the defendant wishes, the judge should spell out as well the forbidden, through-the-box purpose to make clear to jurors how they are not supposed to use the evidence. Close-up of Rule 404(b) other crimes, wrongs, or acts refers to any acts other than those directly at issue in the case. The statement is not admissible to prove the character of a person in order to show action in conformity therewith is superfluous it restates the substance of Rule 404(a) Trial judges are not required to admit evidence of other acts whenever such evidence does not violate the propensity evidence ban. The rule says may . . . be admissible for other purposes. o Typically a judge will weigh the evidence using a 403 balancing analysis and exclude it if its probative value is substantially outweighed by the risk of unfair prejudice, confusion of the issues, or undue delay.

The permitted purposes listed by Rule 404(b) are not exceptions to Rule 404(a). They are merely possible uses of other-acts evidence not banned by Rule 404(a). o Genuine exceptions to Rule 404(a): FRE 404(a)(1) FRE 404(a)(2) FRE 404(a)(3), as elaborated by FRE 607, 608, and 609 FRE 413 FRE 414; and FRE 415

Problem 3.1 Hacker Admissible Evidence in dispute: evidence of a later theft Under Rule 404(b), evidence of a crime, wrong, or other act is not admissible to prove a persons character in order to show that on a particular occasion the person acted in accordance with the character. Here, the prosecution is attempting to show that the defendant has committed another act like this, so he has the propensity to commit such a crime, therefore, he committed this crime; however, the evidence may be admissible to prove knowledge. The prosecutor could use the evidence to show that defendant had the knowledge to commit the charged crime. This was a crime that required specialized knowledge and the evidence shows he has that knowledge. The judge would then do a 403 analysis to determine if the probative value is substantially outweighed by the risk of unfair prejudice. There is a high risk of unfair prejudice, but the probative value is extensive. The risk probably does not substantially outweigh the probative value, so it would not be excluded. Problem 3.2 Drug Seller Inadmissible Evidence in dispute: prior conviction for selling drugs The prosecution would try to bring in the evidence to show that defendant had knowledge of how, where, etc. to conduct a drug transaction. The government has identified an admissible exception; however, the probative value under a 403 analysis seems to be very small, so it would be substantially outweighed by the risk of unfair prejudice. Problem 3.4 U.S. v. Peltier 1 Some admissible Evidence in dispute: outstanding arrest warrant for murder evidence of fleeing an attempted murder wrap. The government has identified a separate purpose other than propensity evidence motive. Peltier not only ran but also fired on the officers and agents. The judge would then do a 403 analysis to determine if the probative value is substantially outweighed by the risk of unfair prejudice. The probative value is high, so the court would probably

admit the evidence with a limiting instruction (i.e., that it could only be used as evidence of a possible motive, not of guilt of the crime at hand.) Problem 3.6 Streetcar Mishap Evidence in dispute: Agents gun with agents thumbprint admitted, ties Peltier to the crime scene (he would have had to be at the crime scene to get the gun); category identity Shell casings that had been fired from the agents revolver admitted, shows consciousness of guilt; he picked up shell casings possibly to clean up the scene Peltiers AR-15 admitted him to the identity of the shooter because it is the same caliber as what shot the agent Coded slips of paper admitted as flight evidence (apply 4-step chain of inference from Meyers to determine whether admission of evidence is proper) o Shows Peltier is conscious of guilt the crimes Hand grenades Fourteen firearms eight with obliterated serial numbers Numerous boxes of shells Tool boxes containing wiring Pocket watches with wires leading out of them Tools, pliers, and empty shell casings Ultimately, all evidence was admitted Problem 3.11 Cleaning his gun Evidence in dispute: defendant shot and killed his first wife he claimed it was an accident, that he had been cleaning his rifle when it accidentally discharged (same scenario as the current charge); he was never charged in connection with the incident. The evidence raises a 404(b) issue because the prosecution is attempting to enter a prior act to show defendant has the propensity to kill his wives. Prosecution is prohibited from submitting the evidence as proof that defendant killed his wife because the same thing happened to his first wife, so he must be guilty of murdering his wife; however, the prosecution may admit evidence of prior wrongs to prove lack of accident. The purpose for admitting the evidence is proper, so the court will conduct a 403 balancing to determine whether the defendant will be unfairly prejudiced and if the prejudice substantially outweighs the probative value. There is high probative value to the evidence showing the similarities between the two deaths. The risk is that the defendant will be unfairly punished because the jury will see the two events and jump to the conclusion that the defendant is guilty without any other proof; however, the probative value is so significant that it is doubtful that the risk of unfair prejudice would substantially outweigh the probative value. As a result, the court will probably allow the evidence.

Res Gestae/Background Evidence Blacks Law Dictionary: o Latin, things done o The events at issue, or other events contemporaneous with them; in evidence law, words and statements about the res gestae are usually admissible under a hearsay exception (such as present sense impression or excited utterance). Where the FRE or state rules fashioned after them are in effect, the use of res gestae is now out of place o The res gestae embraces not only the actual facts of the transaction and the circumstances surrounding it, but the matters immediately antecedent to and having a direct causal connection with it, as well as acts immediately following it and so closely connected with it as to form in reality a part of the occurrence. State v. Fouquette, 221 P.2d 404, 4160417 (Nev. 1950). Refer to Zackowitz o The explanation about the missing holes See also, United States v. Frederick o Where theres an act that raises a 404(b) problem but cant extricate itself from the event, it may be proper

The Huddleston Standard (pp. 190 196) Huddleston v. United States, 485 U.S. 681 Summary: In April 1985, Petitioner was charged with one count of selling stolen goods in interstate commerce, and one count of possessing stolen property in interstate commerce. The two counts related to two portions of a shipment of stolen Memorex video cassette tapes that petitioner was alleged to have possessed and sold. The only issue was whether Petitioner knew the tapes were stolen. Issue: whether the trial court must make a preliminary finding before similar act and other Rule 404(b) evidence is submitted to the jury. Evidence in dispute: evidence of similar acts Testimony of Paul Toney, a record store owner in February 1985, petitioner offered to sell new 12: black and white televisions for $28 apiece; petitioner indicated that he could obtain several thousand of these televisions Testimony of Robert Nelson, an undercover FBI agent posing as a buyer for an appliance store in May 1985, petitioner offered to sell him a large quantity of Amana appliances that were determined to be part of a stolen shipment o Note: the rule just says other acts, not prior acts Arguments: Petitioner said that the he had no knowledge that the items were stolen and that his sales were on a commission basis; that he was not on trial for the other acts but that the admission of the evidence ran the risk of the jury convicting him because of his prior acts.

Petitioner argued that the jury ought not to be exposed to similar act evidence until the trial court has heard the evidence and made a determination under Rule 104(a) tht the defendant committed the similar act. Prosecution said it showed knowledge and the jury was instructed that the evidence was to used only to establish knowledge, and not to prove character.

Holding: Such evidence should be admitted if there is sufficient evidence to support a finding by the jury that the defendant committed the similar act. Rule 104(b) standard preponderance of the evidence Court sited low price, lack of bill of sale, defendants merchandise came from the same supplier Evidence of similar acts is admissible when it satisfies a four-part test: Whether the evidence is offered for a proper purpose (Rule 404(b)) (not propensity) Whether it is relevant for that purpose (Rule 402) Whether its probative value is outweighed by the risk of unfair prejudice (Rule 403), and then Hands out an FRE 105 limiting instruction. Handout #3: Standards of Appellate Review of Evidentiary Issues Was there an error? o Standard usually used is abuse of discretion. o Questions of facts are usually reviewed for clear error Appellate court will usually defer to the trial court on issues of fact. o Questions of law are usually reviewed de novo Appellate court will usually review questions of law without giving deference to the trial court Including constitutional issues was the defendants constitutional rights violated Does the error require reversal of the judgment? o Structural Error o Constitutional harmless error An error that misapplies constitutional protections o Harmless error Rule 103 An error is not harmless when an error affecting a substantial right of a party is an error which had a substantial influence on the outcome or which leaves one in grave doubt as to whether it had such effect. Its a harmless error when inclusion/exclusion of the evidence would not have affected the outcome of the trial. If the trial court made a harmless error, then the decision will not be reversed. o Plain error

Propensity Evidence in Sexual Assault Cases Rule 413 Similar Crimes in Sexual-Assault Cases Rule 414 Similar Crimes in Child-Molestation Cases Rule 415 Similar Acts in Civil Cases Involving Sexual Assault or Child Molestation FRE 413, 414, and 415 are true exceptions to the propensity evidence ban. These three rules allow prosecutors and civil plaintiffs to offer evidence of the defendants other acts of sexual assault or child molestation on any matter to which it is relevant. o The rules save federal prosecutors the effort of having to justify evidence of other sexual offenses in this way. State v. Kirsch, 139 N.H. 647, 662 A.2d 937 (1995). Summary: Following a jury trial, David W. Kirsch (Defendant) was convicted of various sexual offenses committed against minors. At trial, the court allowed testimonial evidence to be heard by the jury of other alleged sexual assaults unrelated to the present charge, and allegedly committed against various other young women. Defendant appeals his conviction here. Issue: Was the testimony from the alleged victims of the uncharged crimes properly admitted under Rule 404(b) as evidence of Defendants motive, intent, and/or common plan or scheme? Rule: New Hampshires equivalent to Federal Rule of Evidence 404(b) prohibits the admission of evidence of other crimes, wrongs, or acts . . . in order to show the person acted in conformity therewith, and only allows evidence of prior bad acts to be admitted when it is offered to prove such things as motive, intent, or a common plan. Facts: Defendant was a member of the Granite State Baptist Church in Salem, and drove the church bus that drove the victims to church; Defendant also attended overnight sleep-overs at the church. Defendant was charged with various sexual assault crimes, based on the allegations of three young girls. At trial, the prosecution attempted to introduce testimony of those three victims, as was the testimony of three other women pursuant to Federal Rule of Evidence 404(b); the testimony of the latter concerned sexual abuse allegedly committed against them by Defendant. The trial court allowed the testimony of the uncharged acts, ruling that the evidence was relevant to prove motive, intent, and common plan or scheme, and therefore was admissible under Rule 404(b). Holding: Reversed and remanded. The testimony concerning the uncharged assaults should not have been admitted, as it constitutes evidence offered to show Defendants propensity to commit sexual assaults and that Defendant acted in conformity therewith, not evidence of Defendants motive, intent, and/or common plan or scheme. Dissent / Concurrence: Justices Thayer and Horton concur in part and dissent in part, arguing that the evidence offered was relevant to show Defendants plan, and should have been admitted.

Discussion: The majority argues that the, ostensible purpose for which the prosecution sought to admit evidence of a multitude of other uncharged sexual assaults was to show the defendants predilection for molesting young females over whom he was able to gain control through engendering trust. At most, this is evidence of the defendants disposition to commit the offenses with which he was charged, impermissible under Rule 404(b). The dissent, on the other hand, argues that, *t+he majoritys narrow reading of the common plan exception essentially requires the State to show the defendants state of mind before he started on his spree of criminal conduct, limiting the exception to a mutually dependent series of events. The dissent cites other New Hampshire precedent to support its contention that, the rule should not be so limited. PROBLEM 3.14 Evidence in dispute: defendants past battering instances against his wife. The evidence is relevant because it makes the probability that defendant was violent towards his wife more likely. However, propensity evidence is not admissible to prove conduct, so the court must determine if the evidence is offered for a permitted purpose. The evidence in dispute is not an exception as defined by Rules 413-415, so you have to look to Rule 404 which states evidence of a persons character is not admissible to probe that on a particular occasion the person acted in accordance with the character or trait; however, evidence of a crime, wrong, or other may be admissible for other purposes such as to prove motive, opportunity, intent, preparation, plan, knowledge, identity, absence of mistake, or lack of accident. The court has stated that a pattern or systematic course of conduct is insufficient to establish a plan. Rather, to be admissible as evidence showing the defendants plan, other bad acts must be constituent parts of some overall scheme. There must be some overall scheme of which each of the crimes is but a part. The prosecution would have a difficult time showing that the previous wrong acts were part of the plan that ends with his wifes death; however, the prosecution may be able to show that the plan was to control his wife, and that plan lead to the death of the victim when the plan failed. This could also be a motive for killing his wife he was unable to control her. The prosecution has identified a permitted use for the character evidence. The court must weigh the probative value of the evidence with the risk of unfair prejudice. The fact that defendant battered his wife has high probative value, but it is also extremely prejudicial against the defendant. The risk is that the jury will convict the defendant based on his past acts rather than for the crime at hand. Given the high probative value, it is unlikely that the court will determine that the risk of unfair prejudice substantially outweighs the probative value. Therefore, the court will probably admit the instances of past wrongs into evidence.

United States v. Guardia, 135 F.3d 1326 Summary: David K. Guardia, M.D. (Defendant) was indicted for two counts of criminal sexual penetration and two counts of battery; the indictment was based on Defendants alleged behaviors

while he was practicing as a gynecologist. The prosecution offered into evidence the testimony of various women who claimed to have been abused by Defendant, and Defendant moved to have the evidence excluded based on the standard of Federal Rule of Evidence 403. The lower court granted Defendants motion, and this appeal followed. Issue: Does the test found in Federal Rule of Evidence 403 (that evidence, otherwise admissible, is inadmissible when the probative value of the evidence is substantially outweighed by the risk of unfair prejudice) apply to evidence introduced under Federal Rule of Evidence 413? Was the lower court correct in excluding the evidence in the present case based on its Rule 403 determination that the risk of prejudice did substantially outweigh the evidences probative value? Rule: In order for evidence to be admissible under Federal Rule of Evidence 413, a defendant must be on trial for, an offense of sexual assault, the evidence must be of and concerning, another offense of . . . sexual assault, the lower court must determine that the evidence is relevant, and the potential prejudicial value of the evidence must not substantially outweigh its probative value. Facts: Defendant was indicted after two women complained that Defendant improperly touched them during gynecological exams; none of the alleged improper behaviors took place in the presence of any additional witnesses. Aside from the alleged victims testimony, the prosecution attempted to introduce, under Federal Rule of Evidence 413, testimony of various other women, who alleged that similar abuses were inflicted on them by Defendant in the past. Defendant moved to have the evidence excluded, based on the standard found in Federal Rule of Evidence 403, and the court granted his motion. Holding: Affirmed. Yes; Federal Rule of Evidence 403 applies to all types of evidence. Yes; the decision to exclude evidence based on Federal Rule of Evidence 403 is within the sound discretion of the trial court, and that court properly exercised its discretion. Discussion: The court reasons that the decision to exclude evidence under Federal Rule of Evidence 403 lies with the court, and when the court determines that the potential for confusion of the issues substantially outweighs the probative value of the evidence offered, such evidence is properly excluded. The court notes its desire to give the benefit of the doubt to the lower court, and states: In this case, the district courts colloquy with the attorneys at the motion hearing and the courts written decision reflect its thoughtful consideration of both the relevance of the Rule 413 evidence and the policies behind Rule 403. Given the deference due district courts in making Rule 403 determinations, we find that the district court did not abuse its discretion in concluding

under Rule 403 that the risk of jury confusion substantially outweighed the probative value of the Rule 413 evidence proffered by the government.

United States v. Mound, 157 F.3d 1153 (8th Cir. 1998), cert. denied 525 U.S. 1089 (1999). Summary: After his conviction for various sexual abuse crimes involving a minor, Alvin Ralph Mound (Defendant) appeals, challenging the trial courts decision to allow into evidence a past conviction of Defendants for child sexual abuse under Federal Rule of Evidence 413. After the lower court admitted evidence of the conviction, the jury convicted Defendant and sentenced him to life imprisonment. Defendant appeals his conviction here. Issue: Is Federal Rule of Evidence 413 unconstitutional as applied because it allows punishment for past acts not currently at issue? Is Federal Rule of Evidence 413 unconstitutional as applied because it violates Defendants equal protection rights? Was it an abuse of discretion for the lower court to admit Defendants prior conviction under Federal Rule of Evidence 413, following a Rule 403 determination? Is it error to admit items into evidence under Rule 413 when the same evidence would be inadmissible under Rule 404(b)? Rule: Courts must apply a Rule 403 test to all evidence offered, and Rule 413, so long as it is, subject to the constraints of Rule 403, is constitutional. Facts: Defendant was accused of sexually abusing his daughter for a period of nearly four years, beginning when she was ten years old. Defendant was convicted by a jury, following a trial, of seven sexual abuse and assault charges, and sentenced to life imprisonment. Prior to the events from which this case stems, Defendant was accused by two other young women of abuse; Defendant pled guilty to the first offense in exchange for the second charge being dropped. Evidence of the conviction of Defendant and evidence relating to the uncharged crime were offered by the prosecution; the lower court admitted into evidence the conviction only, excluding the evidence related to the uncharged crime. Holding: Affirmed. No; Congress has the ultimate power to create exceptions to the Federal Rules of Evidence it creates, and Rule 413 is essentially an exception to the past practice of excluding evidence of prior bad acts. No; the Rule bears a rational relation to some legitimate end and Congresss judgment in enacting the Rule was rational.

No; there were two alleged past acts, and the court properly applied the Rule 403 test and, in its discretion, determined to allow into evidence the prior conviction, but not allow testimony concerning the uncharged crime. No; according to the legislative history of Rule 413, the rule was intended to have the effect of allowing certain evidence that would be inadmissible under Rule 404(b), and the present case is one such situation.

Discussion: The court reasons that the cautionary instruction to the jury guarded against unfair prejudice, and in light of the instruction, the court exercised its discretion in a proper manner. The cautionary instruction reads: o This defendant was convicted in 1988 of sexual abuse of a minor. This does not mean that he is guilty of any of the charges of aggravated sexual abuse or any other offense as to which he has pled not guilty in this case which you will be deciding. You may give such evidence and the testimony of this witness no weight or such weight as you think it is entitled to receive. . . . [T]his evidence is being received for a limited purpose only. Proof of the Defendants and the Victims Character FRE 404(a)(1), 404(a)(2), and 405 Rule 404. Character Evidence, Crimes or Other Acts Subsection (a) prohibited uses Subsection (b) exceptions for defendant or victim in a criminal trial o Remember, these exceptions do not apply to civil cases. Rule 405 Methods of Proving Character describes how character evidence can be brought in. 405(a) on direct, the norm is reputation or opinion only; on cross examination, the court may allow an inquiry into relevant specific instances of the persons conduct Michaelson v. United States, 335 U.S. 469 (1948). Summary: Michaelson was convicted of bribing a federal official. He is claiming entrapment as an affirmative defense. Evidence in dispute: character witnesses cross examinations of in which specific acts were introduced (prior arrest for receiving stolen goods). Rule: Under Rule 405, specific acts can come in on cross examination Holding: There was no legal error; judgment affirmed PROBLEM 3.16. Character of Victim I There is a split between state courts and federal courts on whether specific acts are admissible; federal courts say no; state courts allow it. Rule 404(a)(2)(B) evidence is limited to Evidence of Habit FRE 406 Halloran v. Virginia Chemicals, Inc., 41 N.Y.2d 386 (1977).

Summary: Holding: Rule: Fewer examples of habit go to the weight of the evidence, not the admissibility.

IMPEACHMENT AND CHARACTER FOR TRUTHFULNESS Modes of Impeachment A lawyer impeaches witnesses by casting doubt on the witnesss accuracy or trustworthiness. Thats an error vs. Thats a lie o Character evidence rules impose no constraint on modes of calling a witness mistaken. o Questions about the witnesss eyesight or hearing, passage of time, the witnesss age, or memory do not attack character o As long as the evidence is relevant under Rule 401 and can survive a Rule 403 weighing test, it is admissible Youre lying vs. Youre a liar o Non-character impeachment Contradiction by Conflicting evidence Contradiction by Past Inconsistent Statement Evidence of Bias o Character-Based Impeachment Rule 607 either party may attack a witnesss credibility, including the party that sponsored the witness Rule 608(a) either party may offer evidence of a witnesss character for untruthfulness Rule 608(b) On cross-examination a party may ask a witness about specific instances of the conduct of a witness. . . to attack or support the witness character for truthfulness Rule 609 either party may seek to impeach a witness by showing her past conviction of a sufficiently serious or deceptive crime. Impeachment by Opinion, Reputation and Cross-Examination about Past Lies United States v. Whitmore, 359 F.3d 609 (D.C. Cir. 2004). Summary: Holding: it was an abuse of discretion and not harmless error to not allow cross examination of the police officer. Rule: Notes: Exclusion reviewed for abuse of discretion Standard for overturning harmless error

Article: Credence, Character, and the Rules of Evidence: Seeing Though the Liars Tale Character as a Source of Impeachment In allowing character evidence on the issue of credence, the law postulates several factual axioms: Dishonest people are more likely to lie in any given situation than honest people. The character trait of veracity/mendacity is detectable by casual observers in the community, and the community consensus is accurately transmitted among acquaintances Ordinary people, properly instructed as jurors, will appreciate the distinction between an inference from dishonest character to untruthful testimony and an inference from dishonest character to criminal conduct Problem 4.1 Bar Fight

Character and Credibility: Study Guide (Part I) Fundamentals: (1) FRE 413: Similar offenses in a sexual assault prosecution; (2) FRE 414: Similar offenses in a child molestation prosecution (3) FRE 415: Similar offenses in a civil action concerning sexual assault or child molestation (4) FRE 404(a)(1): Character of accused; (5) FRE 404(a)(2): Character of victim (6) FRE 404(a)(3): Character of witness Rule 404(a)(3) says character evidence may be admitted as provided in Rules 607, 608, and 609. All character evidence, to be relevant, must bear on a pertinent trait of character. o Cant offer evidence of a defendants peaceful disposition for a non-violent crime; it isnt relevant. o Could offer evidence of a defendants peaceful disposition for a violent crime o Evidence of the character of a witness, to be pertinent, must bear on the witnesss character for truth-telling. Rules 608 and 609 say that once a witness has offered testimony, the opposing lawyer may use character evidence to attack the witnesss credibility. The witnesss sponsor may then use character evidence to support her credibility. o These rules constrain when and how a party may offer evidence about a witnesss character for truthfulness

Impeachment Evidence Flowchart Rule 608(a) permits a litigant to offer evidence of a witnesss character for truthfulness or untruthfulness in the form of opinion or reputation. Rule 608(b) permits a party to ask on cross-examination about specific instances of the conduct of a witness. o Rule 608(b)(1) and (b)(2) address two separate contexts in which such inquiry may be made During cross-examination of Witness Jones (principal witness)and During cross-examination of a character witness who has offered testimony about Witness Joness character for truthfulness or untruthfulness. o Rule 608 allows more liberal use of specific instances of conduct than does Rule 405(a) which governs the forms of character evidence offered under Rules 404(a)(1) and 404(a)(2). o Rule 405(a) does not permit the prosecutor to cross-examine a witness about a previous incident. There are two exception in which the prosecutor might be permitted to question the defendant about the old incident:

The prosecutor could argue that she is offering the evidence for a non-propensity purpose under Rule 404(b) If the defendant has vouched for his own character, then the defendant arguably has made himself his own character witness, and the question therefore may be proper under Rule 405(a)

Limitations on FRE 608(b)(1) Two explicit limitations: o The specific instance of conduct must be probative of truthfulness or untruthfulness o It may not be proved by extrinsic evidence Other (less explicit) limitations o The inquiry must survive a Rule 403 weighing test o A lawyer may not ethically ask about specific instances of conduct without having a good-faith basis for believing that they took place o The trial judge must exercise reasonable control to protect witnesses from harassment or undue embarrassment o Rule 608(b) is not a back door for admission of evidence not admissible under Rule 609 Many courts will not permit a lawyer to use Rule 608(b) to offer evidence that is of a type covered by Rule 609, but is also excluded by Rule 609 For example, if a lawyer tried to use an eleven-year-old conviction, excluded by Rule 609(b), as a specific instance of conduct under Rule 608(b), many courts would exclude the evidence.

Impeachment with Past Convictions FRE 609 Used only to show evidence of untruthfulness, not to show the propensity to commit a certain type of crime.

Prior Conviction Felony (punishable by death or more than one year)

Other Witnesses 403 probative value is substantially outweighed by the risk of unfair prejudice 609(a)(1)(A) Not admissible

Misdemeanor , generally (punishable by less than one year) Crimes of falsity (can be a misdemeanor) Rule 609(a)(2) Old (> 10 years ago)

Criminal Defendant 50/50 Bal. probative value vs. unfair prejudice (5 factors in Brewer) 609(1)(B) ties go to exclusion of evidence; favors inadmissibility for crimes that are similar Not admissible

Admissible dont even have to do a Rule 403 balancing Reverse 403 tie goes to exclusion; probative value has to substantially outweigh the risk of unfair prejudice Admissible only if necessary for a fair trial the highest standard to meet 609(d) (for example, comes in on cross-examination of a government witness)

Admissible dont even have to do a Rule 403 balancing Reverse 403: - 609(b)

Juvenile adjudication

Not admissible 609(d)

PROBLEM 4.3 At his trial on charges of grand theft in 2007, the defendant took the stand and testified that he was hundreds of miles away at the time of the crime. The prosecutor then attempted each of the following modes of impeachment: Prosecutor asked the defendant on cross-examination whether he shot a man in the arm during a drunken barroom brawl in 2001. o Inadmissible under 609 because its not a prior conviction. o Inadmissible under Rule 608(b) because it is not probative of truthfulness or untruthfulness. She called a witness to testify that he saw the defendant shoot a man in the arm during a drunken barroom brawl in 2001. o Inadmissible under 609 because there was no conviction. o If the question is posed on direct examination, Rule 608(a) limits the prosecutor to opinion or reputation evidence. Here, she asked about specific acts. Moreover, rule 608(a) limits the prosecutor to questions bearing on character for truthfulness or untruthfulness.

She offered evidence that the defendant was convicted of assault and battery by means of a dangerous weapon and sentenced to serve five years in state prison for having shot a man in the arm during a drunken barroom brawl in 2001. o Maybe admissible. Rule 609(a)(1) will permit this question if the court determines that the probative value of admitting this evidence outweighs its prejudicial effect to the accused. The probative value of a six-year-old crime of violence in showing the defendants character for untruthfulness is perhaps not as great. The crime is not particularly recent, and it is unclear how a violent act bears on truthfulness. On the other hand, any shooting is a very serious crime. And the risk of unfair prejudice to the defendant is moderated by the dissimilarity of the old crime (involving violence) to the present charges (alleging theft). She asked the defendant on cross-examination whether he was convicted of turnstile jumping and sentenced to three months in county jail in 2001. o Probably inadmissible. If turnstile jumping is punishable by more than one year in prison, which seems unlikely, and if the judge finds that the probative value of the evidence outweighs the danger of unfair prejudice, the evidence may be admitted under Rule 609(a)(1). If either of these conditions is not met, the only possible routes of admission are Rules 609(a)(2) and 608(b). Rule 609(a)(2) doesnt apply because turnstile jumping does not involve false statement. And most judges would not regard a conviction of turnstile jumping to be probative of truthfulness or untruthfulness, as required by Rule 608(b). Moreover, since the evidence is of a type governed by Rule 609, yet is excluded by the terms of Rule 609, yet is excluded by the terms of Rule 609 (if not punishable by more than a year), many judges would not permit a back-door admission under Rule 608(b). Even if the evidence were admitted under Rule 608(b), there still would be the problem of getting past Rule 403. She offered evidence that the defendant was convicted of lying to a federal investigator and sentenced to two years in a federal penitentiary in 1993. o Possibly admissible. Because of the age of the conviction, it is presumptively inadmissible under Rule 609(b): it is not admissible unless the judge finds that its probative value substantially outweighs its prejudicial effect (and unless the notice provisions of Rule 609(b) have been met). Still, the evidence seems rather probative if not for its age, the conviction of lying to a federal investigator would have been automatically admissible under Rule 609(a)(2). And the evidence poses only a moderate risk of unfair prejudice because the present charges of theft bear little similarity to the old conviction of lying to an investigator. Some judges would take the defendants more recent criminal record into account. If the defendant recently has been involved in other criminal matters, this old conviction may be more probative of his current character for truthfulness than if he has been out of trouble for a while.

United States v. Brewer, 451 Supp. 50 (E.D. Tenn. 1978). Summary: Brewer was charged with one count of kidnapping and one count of transporting a stolen motor vehicle. He motioned to have evidence of past crimes suppressed if he took the stand to testify.

Evidence in dispute: Four past convictions kidnapping, rape, aggravated assault, and assault with a deadly weapon Plaintiff: 1) the evidence is out of the ten-year limit; 2) the evidence fails the balancing test of Rule 609 Prosecution: 1) the ten-year limit starts when defendants sentence is over (released from confinement), not when the sentence was given; 2) the evidence is admissible to impeach the defendants testimony Rule: factors to use by the Courts to determine whether probative value of admitting the evidence outweighs its prejudicial effect under an FRE 609(a) balancing: 1) The nature of the crime; 2) The time of conviction and the witness subsequent history; 3) Similarity between the past crime and the charged crime; 4) Importance of defendants testimony; and 5) The centrality of the credibility issue Holding: The probative value of the prior kidnapping conviction on the issue of defendants truthfulness does not outweigh the prejudicial effect knowledge of such conviction could have on the jury. Admission of the other three convictions, all involving serious crimes, should sufficiently serve the purpose of impeaching the defendants credibility.

Character and Credibility: Study Guide (Part II) Rule 609 permits a litigant to impeach a witness with evidence that the witness has been convicted previously of a crime. o Such evidence is usually presented during the cross-examination of the witness. o The evidence also may take the form of a court document reflecting the conviction. Inquiry into the underlying details of the crime generally is not permitted. Rule 609(a)(1) o Is written in terms of available (not actual) penalty o The one year available penalty means, generally, that the crime was a felony o Evidence offered against the accused in a criminal case must survive a stricter weighing test: it is admitted only if its probative value outweighs its potential to cause unfair prejudice to the defendant. o Five factors that generally govern this weighing test: The nature of the crime The time of conviction and the witnesss subsequent history; Similarity between the past crime and the charged crime; The importance of the defendants testimony; and The more critical the defendants testimony is to his case, the more hesitant the court should be to admit the impeaching evidence The centrality of the credibility issue.

Trial judges have discretion under Rule 403 to admit evidence of a past conviction while withholding from jurors the nature of that conviction. In general, trial courts allow jurors to learn the nature of the past crime.

Rule 609(a)(2) Carved out a class of convictions as particularly probative of untruthful character and declares that they shall (must) be admitted regardless of punishment. o Admission is not within the discretion of the trial court. Listed crimes: o crimes such as perjury, false statement, criminal fraud, embezzlement, or false pretense, or any other offense in the nature of crimen falsi, the commission of which involves some element of deceit, untruthfulness, or falsification bearing on the [witnesss] propensity to testify truthfully. The rule applies only if it readily can be determined that establishing the elements of the crime required proof or admission of an act of dishonesty or false statement by the witness. Examples of sources that readily could support such a finding: o The statutory elements of the charged crime; o The face of the trial courts judgment; o The indictment; o A statement of admitted facts (if any); or

o Rule 609(b)

Jury instructions (if any)

When the conviction is more than ten years old, it will be excluded unless the court determines that the probative value of the conviction substantially outweighs its prejudicial effect. o Establishes a rebuttable presumption that evidence of old convictions is not admissible.

Rule 609(d) Juvenile adjudications are never admissible in civil cases or to impeach the testimony of criminal defendants. Even when used against other witnesses in a criminal case, they must survive the strictest standard of any prescribed in these rules. o They are admissible only if the offense would be admissible to attack the credibility of an adult and the court is satisfied that admission is necessary for a fair determination of the issue of guilt or innocence.

The Various Standards of Admission The rules drafters appear to have intended the various subparts of Rule 609 to reflect a number of discrete Rule 403-style weighing tests. Standards from most permissive to most restrictive: o 609(a)(2): conviction of a crime requiring proof or admission of an act of dishonesty or false statement shall be admitted, subject only to the limits imposed by Rules 609(b), (c) o 609(a)(1): if the witness is not the accused in a criminal case, conviction of a crime punishable by death or imprisonment in excess of one year. . . shall be admitted, subject to Rule 403. o 609(a)(1): if the witness is the accused in a criminal case, conviction of a crime punishable by death or imprisonment in excess of one year. . . shall be admitted if the court determines that the probative value of admitting this evidence outweighs its prejudicial effect to the accused. o Rule 609(b): if more than ten years has elapsed since the date of the conviction or of the release of the witness from confinement, whoever is later, evidence of a conviction is not admissible. . . unless the court determines, in the interests of justice, that the probative value of the conviction . . . substantially outweighs it prejudicial effect. o Rule 609(d): Evidence of a juvenile adjudication is never admissible in a civil case or to impeach the accused in a criminal case, but may be admitted to impeach another witness in a criminal case if the evidence otherwise qualifies under Rule 609 and the court is satisfied that admission in evidence is necessary for a fair determination of the issue of guilt or innocence. Judgments about the varying probativeness of the evidence they govern o Rule 609(a)(1): More serious crimes suggest readiness to lie under oath. This rule therefore generally requires that crimes used to impeach have been punishable by more than a year in prison.

o o o o

Rule 609(a)(2): Crimes involving deceit are especially probative of ones propensity to lie and are therefore made automatically admissible Rule 609(b): More recent crimes are more probative of ones present character. So this rule makes it far harder to admit evidence of older convictions. Rule 609(c): a persons successful rehabilitation diminishes the probativeness of past crimes, and a later finding of innocence would reduce probativeness to near zero. Rule 609(d): If the witness was a juvenile at the time of her past offense, there is a greater chance that her character has substantially altered (and improved) in the interim.

THE RAPE SHIELD LAW Historical Backdrop People v. Abbot, 19 Wend. (N.Y.) 192 (1838). Summary: rape case in which the womans sexual history was allowed o the question to the prosecutrix herself, whether she had not had previous criminal connection with other men, was, I think, proper. . . o a mixed case will not do; the connection must be absolutely against the will; and are we to be told that previous prostitution shall not make one among those circumstances which raise a doubt of assent? that the triers should be advised to make no distinction in their minds between the virgin and a tenant of the stew? between one who would prefer death to pollution, and another who, incited by lust and lucre, daily offers her person to the indiscriminate embraces of the other sex? Evidence in dispute: rape victims past sexual history (she was a prostitute) Holding: The question was proper. Notes: Wigmore treatise Women lie. No judge should ever let a sex-offense charge go to the jury unless the female complainants social history and mental makeup have been examined and testified to by a qualified physician. Overview of Rule FRE 412 Threshold issue must be a civil or criminal case Two types of prohibited uses: Evidence offered to prove that a victim engaged in other sexual behavior; or o Looks like 608(a) Evidence offered to prove a victims sexual predisposition Criminal Cases 412(b)(1)

Civil Cases 412(b)(2)

HEARSAY Defining Hearsay FRE 801 (a) Statement means a persons oral assertion, written assertion, or nonverbal conduct, if the person intended it as an assertion. (b) Declarant means the person who made the statement.

(c) Hearsay is a statement, other than one made by the declarant while testifying at the trial or hearing, offered in evidence by a litigant to prove the truth of the matter asserted by the declarant. (1) The declarant does not make while testifying at the current trial or hearing; and (2) A party offers in evidence to prove the truth of the matter asserted in the statement. Notes: If the declarant is on the stand, the statement is not hearsay If the declarant is testifying about a statement that he made out-of-court, it is hearsay, if used to prove the truth of the matter asserted in the statement. (see Problem 7.3) FRE 802 Hearsay is not admissible unless any of the following provides otherwise: A federal statute These rules; or Other rules prescribed by the Supreme Court Nonhearsay Uses of Out-of-Court Statements To prove the impact of the statement on someone who heard it; To prove a legal right or duty that was triggered by or an offense that was caused by uttering the statement; and To impeach the declarants later, in-court testimony. To prove the impact of the statement on someone who heard it The rule does not exclude as hearsay those statements offered only to show their effect on the listener. For example, in a trial where defendant is claiming self-defense, defendant wants to testify that someone warned him that the victim had said he had a gun and was looking for the defendant. The statement is not hearsay if offered to prove that he had reason to fear the victim at the time of the attack. The statement is not offered to prove the truth of the assertion, only that the there was a reason to fear the victim. Legally Operative Words (Verbal Acts): Verbal acts have legal force independent of the speakers intended meaning. Examples of verbal acts: o If other conditions are met, saying I accept after an offer can create a contract. o Saying I do at the altar makes a marriage. o Saying I will kill you constitutes a threat. o Saying Hes a thief in public can be slander.

The statements operate independently of the speakers belief or intended meaning. The soundness of the witnesss testimonial capacities therefore doesnt matter, so there is no reason to exclude her words as hearsay.

Inconsistent Statement Offered to Impeach The theory is not that the out-of-court statement is true, so the present testimony must be false. Instead, prior inconsistent statements prove that the witness has said different things at different times about this fact, so the testimony on this point cant be trusted. The truth of the out-of-court statement is irrelevant. Evidence of inconsistent statements offered only to impeach therefore is not hearsay. Two Main Questions to Ask in Determining Hearsay: Is the litigant offering the statement to prove the truth of what it says or was meant to say? o Example: is the litigant offering the statement John shot Sam to prove that John killed or indeed did shoot Sam? Did the declarant assert (i.e., intend to communicate) that fact? o If yes, to both questions, the statement is hearsay and is inadmissible pursuant to FRE 802 unless an exception applies. o If not, the statement is not hearsay and is admissible. The judge may give a limiting instruction according to FRE 105 confining jurors to the permitted, nonhearsay use of the words Notes: Examine the assertion and possible non-assertions in context. o A trial judge must examine all the facts contextually to determine whether the conduct in question was intended as an assertion. Close cases should be resolved in favor of admissibility.

PROBLEM 7.1 Affidavit Out-of-court statement: the SUVs flunked the safety test Made by a witness to the rollover crash test (she died before the trial, so she was unable to appear in court) Purpose: to assert that the automaker knew about the rollover hazard Its hearsay. (would now move to exceptions and balancing under FRE 403) PROBLEM 7.2 Gesture Out of court sometime in March Statement (gesture) intending to assert declarant was short on cash in March Purpose to show he was short on cash in March

PROBLEM 7.3 QUOTING HERSELF Out of Court at the police station Statement the man who robbed her was number three in the lineup Purpose to show that he was the man who robbed her. Hearsay, but not really. Nonhearsay statements are listed in FRE 801(d)(C) Defining Assertions FRE 801(c) hearsay is an out-of-court statement offered to prove the truth of the matter asserted. FRE 801(a) a statement is an oral or written assertion or nonverbal conduct if intended to as an assertion. Where evidence of non-verbal conduct is relevant only as supporting inferences from the conduct to the belief of the actor and thence to the truth of his belief, prevailing doctrine stigmatizes the evidence as hearsay, inadmissible unless accommodated within one of the exceptions to the rule. (a see-do rule) Non-assertive conduct is excluded from the definition of hearsay. A statement (so as to be subject to the hearsay ban) would include . . . non-verbal conduct of a person [only when] intended by him as a substitute for words in expressing the matter stated. o Conduct must be taken as freed from at least one of the hearsay dangers, mendacity Mendacity = the quality of untruthfulness A man does not lie to himself. If the conduct is for the person doing the acting, and not as a show for others, then it is assumed that the conduct is trustworthy (whether the actor is an egregious liar or a paradigm of veracity). Accordingly, the lack of opportunity for cross-examination in relation to his veracity or lack of it, would seem to be of no substantial importance. Statements of Party-Opponents Although the Constitution protects against coerced confessions, the hearsay rule poses no obstacle to confessions made freely. Rule 801(d)(2)(A) says that a partys own words are not hearsay when offered against her at trial. Rationales: o Most statements offered by a litigant against an opponent were against the opponents interests when she made them. When a prosecutor offers a defendants own words against her, usually it is because those words incriminate the defendant. Although people often lie to advance their interests, people rarely lie in ways that hurt themselves. o Cross-examination The inability to cross-examine hearsay declarants is a principal justification for the hearsay rule. A defendant cant complain that she cant cross-examine the declarant when she is the declarant.

Adversary system The advisory committee tells us that *a+dmissions by a party-opponent are excluded from the category of hearsay on the theory that their admissibility is the result of the adversary system. Any statement that a party makes may come in against her because she said it. She will not be heard to complain that her own statement may be unreliable. Page 387 problems: Problem 5: Not hearsay because the call reflects the belief of the caller that this is a gambling den, but the statement is not meant to communicate that it is a gambling den because everybody already knew that. The caller was not intending to assert that the establishment is a gambling den. Problem 6: is the utterance an assertion? Hearsay; even though the form of the assertion was a question, (why did you stab me?)Yankel was asserting that Nelson was the man who stabbed him. It was meant to identify the stabber and is being offered to prove that Nelson was the stabber. Keep in mind that even though its hearsay, it might possibly be brought in under the dying declaration exception to hearsay. It would not come in under the prior identification exception. Hearsay Quiz 2. Declarant convenience store owner; the statement asserts that the driver of the fleeing car robbed the store and the prosecution is using it to prove that the driver of the fleeing car robbed the store. (the statement (the driver of the fleeing car robbed the store) is being used to prove the truth of the matter that was asserted (that the driver of the fleeing car robbed the store)). 3. Not hearsay the officers testimony is given in court and there is no statement (the store owner chasing the man was not intended as an assertion) 7. Not hearsay testimony offered in court as to his own experience (Note: even if it were, it would still come in as a statement by a party-opponent.) 9. Hearsay her statement (I never rang up a single sale all night) is offered to prove that there was no one in the store. Its an indirect assertion. Were relying on her testimonial capacities. 10. Not hearsay the testimony that there were no other robbery calls is actually a statement about the absence of a statement. There must be a statement in order to have hearsay. 13. Not hearsay not being used to prove his name is Jeffrey Small. Its being used to show the defendant lied. 16. Not hearsay the statement (I wrecked the car today, and the insurance agent says they wont cover it) is not being offered to prove that she didnt have insurance; being offered to prove motive.

Exceptions to the Hearsay Rule: An Introduction Nonhearsay 1) Federal rule 801(d) a. Four types of statements which by definition are nonhearsay. i. Prior inconsistent statements (FRE 801(d)(1)) declarant must be available and subject to cross-examination regarding the statement 1. Rule: prior sworn inconsistent statements are admissible both substantively and to impeach. If the statement is not sworn, only admissible to impeach. 2. Sworn = subject to perjury proceedings 3. Affidavits are not sworn at a proceeding, so they are only admissible to impeach. ii. Prior consistent statements (FRE 801(d)(1)) declarant must be available and subject to cross-examination regarding the statement 1. Nonhearsay where offered to rebut a charge of recent fabrication or improper influence. Admitted substantively. iii. Prior identifications (FRE 801(d)(1)) declarant must be available and subject to cross-examination regarding the statement 1. Rule: a prior statement of identification of a person made after perceiving him. 2. Admissible substantively iv. Admissions (FRE 801(d)(2)) 1. Five types: a. Direct admission (general admission) b. Admission by conduct or silence c. An authorized admission d. A vicarious admission e. A co-conspirators admission 2. Definition: 2) Verbal acts 3) Nonassertive conduct 4) State of mind

The Partys Own Words FRE 801(d)(2)(A) Problem 7.11 Billables Nonhearsay 1) statement by a party-opponent; 2) business records exception the billables are her own records that she was able to work. Problem 7.12 Take my blood The statement should be admitted as statements of party-opponents. Adoptive Admissions FRE 801(d)(2)(B) Problem 7.13 Buddies The statement is offered against an opposing party and is one the party manifested that it adopted or believed to be true.

The buddy went to the bench and took out a baggie of drugs after she made her statement.

Statements of Agents FRE 801(d)(2)(C) and (D) Mahlandt v. Wild Canid Survival & Research Center, Inc., 588 F.2d 626 (8th Cir. 1978). Summary: Three-year-old boy is severely hurt. He is found in the neighbors yard. The neighbor has a wolf, and the center is being sued for the injuries. The wolf belonged to the center. It was the home of Mr. Poos. Testimony in dispute as hearsay: 1) Contents of a note Mr. Poos taped to the door of Owen Sexton, President of Wild Canid Survival and Research Center, Inc. 2) Statement Mr. Poos made to Mr. Sexton later in the day that Sophie had bit a child that day 3) An abstract of minutes from a meeting held in which there was a great deal of discussion about the incident. Holding: 1) Not hearsay, admissible against Mr. Poos and the center. It was his statement, and as such was clearly different from the reported statement of another. Also, he was an agent of the center, so its in. 2) Not hearsay, admissible against Mr. Poos and the center. It was his statement, and as such was clearly different from the reported statement of another. Also, he was an agent of the center, so its in. 3) Not hearsay and admissible against the Center a. The directors as primary officers of the corporation had the authority to include their conclusions in the record of the meeting. So the evidence would fall within 801(d)(2)(C) as to Wild Canid Survival and Research Center, Inc. But hearsay, not admissible against Mr. Poos, b. There was no servant, or agency, relationship which justified admitting the evidence of the board minutes as against Mr. Poos. None of the conditions of 801(d)(2) cover the claim that minutes of a corporate board meeting can be used against a non-attending, nonparticipating employee of that corporation. Problem 7.15 Coconspirators Statements FRE 801(d)(2)(e) and 104(a) Bourjaily v. United States, 483 U.S. 171 (1987). Summary: Bourjaily was charged with conspiring to distribute cocaine and possession of cocaine with intent to distribute. Testimony in dispute: coconspirators statement that Bourjaily was involved used to prove the conspiracy. Conflict: Glasser bootstrapping rule vs. FRE

bootstrapping rule: Coconspirators statements are admissible over the objection of an alleged co-conspirator, who was not present when they were made, only if there is proof aliunde that he is connected with the conspiracy. . . Otherwise, hearsay would lift itself by its own bootstraps to the level of competent evidence. Under Glasser, the court could not look at the hearsay statements themselves for their evidentiary value. You couldnt use the statement of an alleged conspirator at all in determining preliminary facts. But, that was before the FRE were adopted, and the FRE superseded Glasser. That means that in making a preliminary factual determination under Rule 801(d)(2)(E), a court may examine the hearsay statements sought to be admitted. Rule: in making a preliminary factual determination under Rule 801(d)(2)(E), a court may examine the hearsay statements sought to be admitted. Three conditions to meet before the coconspirators exception applies: A conspiracy existed The conspiracy involved the person against whom the statement is offered And that the statement(s) were made during and in furtherance of the conspiracy. Holding: Lonardos out-of-court statements were properly admitted against Bourjaily. Afterthoughts: The theory of the coconspirator exception The agency relationship The existence of a conspiracy In its report on Rule 801(d)(2)(E), the Senate Judiciary Committee noted it is this committees understanding that the rule is meant to carry forward the universally accepted doctrine that a joint venturer is considered as a coconspirator fr the purposes of this rule even though no conspiracy has been charged. Distinguishing 104(a) Questions from 104(b) Questions Rule 104(a) relevance in general The court must decide any preliminary question about whether a witness is qualified, a privilege exists, or evidence is admissible. In so deciding, the court is not bound by evidence rules, except those on privilege. By a preponderance of the evidence Rule 104(b) conditional relevance When the relevance of evidence depends on whether a fact exists, proof must be introduced sufficient to support a finding that the fact does exist. The court may admit the proposed evidence on the condition that the proof be introduced later.

Standard: sufficient evidence; whether a reasonable jury could find the conditional fact by a preponderance of the evidence. Two facts about the dual standards of proof: The preponderance standard of Rule 104(a) is higher than the sufficient-evidence standard of 104(b) o Both 104(a) and the Courts interpretation of that rule in Bourjaily make clear that the evidence used to prove facts under that rule need not itself be admissible. Under Rule 104(b), however, only admissible evidence may be used to prove contested preliminary facts. o The rule requires the introduction of evidence sufficient to support a finding of the fulfillment of the condition. Only admissible evidence may be introduced. So, basically, rule 104(b) governs matters of conditional relevance; Rule 104(a) governs everything else. Rationalizing the distinction: Judges and Jurys Separate Spheres?

Problem 7.16 Y is a party-opponent, so no hearsay problem here. BUT, N was the translator, so the admissibility of the statement of how pure the heroine was depends on the admissibility of the different layers. N, as translator, was the spokesperson of Y. Admissible under 801d)(2)(C) o Must show by a preponderance of the evidence that N was acting under the authorization of Y. (Rule 104) Could also (possibly) get it in under 801(2)(E) o Must show that there was a conspiracy between Y and N, the statement was made during and in furtherance of the conspiracy. Everything happened the way it was explained to the agent seeking to buy drugs. Strong evidence that there was a conspiracy. Past Statement of Witnesses and Past Testimony Introduction Under Rule 613, past inconsistent statements, when offered to impeach, are not offered for the truth of what they assert, but merely to show that the witness says different things at different times and therefore should not be believed. o The statements do not come in substantively as proof of what it says so it is not hearsay, and the judge may give a Rule 105 limiting instruction. Under Rule 801(d)(1)(A), the statements do come in substantively, and the jury may consider them for their truth. o A litigant must satisfy far more stringent conditions to admit a past inconsistent statement for its truth than to impeach.

Inconsistent Statements Offered to Impeach United States v. Barrett, 539 F.2d 244 (1st Cir. 1976). Summary: Testimony in dispute: Holding: Problem 7.19 A prior inconsistent statement given under oath can be offered substantively at trial. Here, F.Ts testimony was given under oath at a grand jury hearing. Therefore, it can be brought in substantively. It could also be brought in to impeach under FRE Rule 608. If she denies her testimony, then the prosecutor could introduce the transcript under FRE 613 as long as she is given the opportunity to testify and explain the discrepancy. The court might allow the prior testimony to convict the defendant of guilt. It depends on the defenses evidence, the ability of the defense to impeach F.T., etc. Problem 7.20 A prior inconsistent statement given under oath can be offered substantively at trial as long as the declarant testifies at the trial or hearing and is subject to cross-examination concerning the statement. Here, F.Ts testimony was given under oath at a grand jury hearing. The prior testimony was given under oath and conflicts with what is now being offered. It is inconsistent testimony; therefore, it appears that the testimony can be brought in substantively. However, the defense will argue that the declarant is not available because she does not remember the event and cannot be effectively cross-examined; therefore, her testimony does not fit within the hearsay exception and is inadmissible. The defense will also argue that the testimony is not inconsistent because it is no testimony at all. It could also be brought in to impeach under FRE Rule 608.

Past Consistent Statements Tome v. United States Summary: A six-year-old girl accused her father of sexual abuse. There were seven statements admitted that implicated her father. Her fathers defense was that the girl wanted to live with her mother (he was awarded custody). The court held that the statements from the witnesses were inadmissible hearsay because they were made after the alleged motive to fabricate arose. Rule: FRE Rule 801(d)(1)(B) permits the introduction of a defendants consistent out-of-court statements to rebut a charge of recent fabrication or improper influence or motive only when those statements were made before the charged recent fabrication or improper influence or motive.

A prior consistent statement has no relevancy to refute the charge unless the consistent statement was made before the source of the bias, interest, influence or incapacity originated.

Holding: judgment of the Court of Appeals for the Tenth Circuit is reversed, and the case is remanded for further proceedings consistent with the opinion. Look at the exception for medical records. This might have been an option for getting the hearsay admitted even though it wasnt an issue here. Statements of Identification Commonwealth v. Weichell (1984) Summary: John Foley and 2 friends were at a park when heard four bangs and saw a man run out of the parking lot. Foley got a brief look at defendant, Weichell and subsequently the police had a composite made up from Foleys description. Dispute: Defense is trying to exclude the evidence as inadmissible hearsay. Rule: a statement o f prior identification is not hearsay if made by a witness who testifies at trial and is subject to cross-examination concerning it. Holding: there is no logical reason to permit the introduction of a witnesss out-of-court identification and to exclude statements identifying the various physical characteristics of a person perceived by the witness, or the composite of all those physical characteristics, which is no more than the sum of the parts perceived. The Court did not answer the question of whether a composite sketch is a statement as defined by Rule 801(a). (Dissent) Liacos, J: in the absence of evidence demonstrating their reliability, we should hold that composite drawings are inadmissible as probative evidence of guilt . . . and reverse the conviction. Exceptions that Require Declarant to be Unavailable Two step analysis: 1. Is the declarant unavailable? Five conditions under FRE 804(a) a. The declarant is exempted from testifying about the subject matter of the declarants statement because the court rules that a privilege applies; b. The declarant refuses to testify about the subject matter despite a court order to do so; c. The declarant testifies to not remembering the subject matter; d. The declarant cannot be preset or testify at the trial or hearing because of death or a then-existing infirmity, physical illness, or mental illness; or e. The declarant is absent from the trial or hearing and the statements proponent has not been able, by process or other reasonable means, to procure:

i. The declarants attendance, in the case of a hearsay exception under Rule 804(b)(1) or (6); or ii. The declarants attendance or testimony, in the case of a hearsay exception under Rule 804(b)(2), (3), or (4). 2. If so, does declarants statement qualify under any of the hearsay exceptions in 804(b)? a. Former testimony b. Statement Under the Belief of Imminent Death c. Statement against interest d. Statement of Personal or Family History e. Other exceptions (transferred to FRE 807) f. Statement Offered Against a Party That Wrongfully Caused the Declarants Unavailability Past Testimony PROBLEM 7.22 Evidence in dispute: testimony at a grand jury trial given under oath

Lloyd v. Am. Export Lines Summary: where a crewman sued his shipping company employer for injuries sustained in a fight aboard ship. Shipping company says they should have been allowed to enter prior Coast Guard hearing testimony. Testimony in dispute: Lloyds testimony in the form of a transcript of a Coast Guard hearing. The prior testimony was regarding a prior assault on Alvarez and Lloyds failure to perform his duties due to intoxication. Rule: In order for the hearsay exceptions of Rule 804 to apply, it is required that the declarant be unavailable that he be absent from the hearing and the proponent of his statement be unable to procure his attendance . . . by process or other reasonable means. Rule 804(a)(5) If it appears that in the former suit a party having a like motive to cross-examine about the same matters as the present party would have, was accorded an adequate opportunity for such examination, the testimony may be received against the present party. Holding: Lloyd was unavailable for purposes of rule 804. Court found that the Coast Guard and Lloyd had similar interests and all the parties had lawyers present at the Coast Guard hearing; so, it was allowed in. Statements Against Interest Under the common law, the statement against interest had to result in pecuniary loss or criminal liability.

In the past, Rule 804(b)(3)(B) only applied to exculpatory evidence. Now it applies even-handedly to both sides. Burden on defendant to prove the trustworthiness of the admission.

Williamson v. United States, 512 U.S. 594 (1994). Summary: Harris was arrested carrying two suitcases of cocaine in the trunk of his rental car. Upon being questioned, he first told authorities that he was going to make a delivery to Williamson by trash dumpster, and then changed his story to say that Williamson was traveling with him in another car. Both stories implicated Williamson. Issue: Whether collateral statements made along with a statement that is against a declarants interest are admissible. Holding: No Rule: Majority Reasoning The policy behind the rule is that statements against a persons interest would not be made by a reasonable person unless he believed them to be true. Collateral statements that are either neutral or self-serving do not have the same guarantee of trustworthiness. Thus, they should be excluded. The ACN notes notwithstanding, all collateral statements are inadmissible because Congress intended the rule to be narrowly applied consistently with the underlying policy for excluding hearsay. Statement as used in the rule does not encompass an entire narrative, but should be narrowly construed to include only those portions that are themselves against interest. Note that this could include statements that incuplate others or exculpate others if they are used to show the existence of a conspiracy. The Court in this case is concerned that the hearsay does nothing to inculpate the speaker; therefore they are less trustworthy. Dissent Reasoning [Kennedy] Excluding collateral statements altogether eviscerates the rule, because most statements against interest are not directly against interest. The court should admit those statements that are facially neutral, but tend to be against the interest of the declarant when viewed as a whole. Break-down of the justices: Ginsburg +3 : none comes in OConnor & Scalia: only the self-inculpitory parts come in (narrow) Kennedy, Rehnquist, Thomas: All of it comes in. Dying Declarations FRE 804(b)(2) Can only be used in a civil case or in a prosecution for homicide. The declarant must believe his death is certain and imminent. The statement must be made about the crime. Shepard v. United States, 290 U.S. 96 (1933). Facts and Procedural History

Dr. Shepard, was charged with the poisoning of his wife who was also his patient. At his trial, the court permitted the prosecution to introduce testimony of a nurse who attended Mrs. Shepard shortly before her death. The nurse testified that Mrs. Shepard asked her to fetch a bottle of whisky from the doctors room. The nurse said the wife had drunk from this bottle before collapsing, requested a test, insisted that the smell and taste were strange, and added, Dr. Shepard has poisoned me. The conviction in Federal District Court was sustained by the Circuit Court of Appeals for the Tenth Circuit. The U.S. Supreme Court granted certiorari. Issue Was the statement Dr. Shepard poisoned me admissible as a dying declaration? Holding: No. Rule: To make out a dying declaration the declarant must have spoken without hope of recovery and in the shadow of impending death (certain and imminent death). Afterthoughts: Shepard v. United States, 290 U.S. 96 (1933). (Part II) Issue Though incompetent as dying declarations, (no evidence that she believed death was imminent), are the mere statements made to the nurse admissible, either as (1) an indication of a state of mind inconsistent with the presence of suicidal intent, or (2) as evidence of present sense impression to prove that Mrs. Shepard had observed the matter remembered? Holding No. Reversed and remanded. Justice Cardozo delivered the opinion of the Court. Reasoning 1. No SOM exception not accepted. Since the testimony had been offered and received as proof of a dying declaration, the government may not now argue an appeal that the declarations were offered to show a persistency of a will to live. Because of the stated purpose of the testimony, was put off his guard. It would now be unfair for the government to shift its ground. The purpose for which normally hearsay evidence is sought to be admitted must be made clear at the time it is introduced. 2. No present sense impression exception not accepted. The government did not use the declarations by Mrs. Shepard to prove her present thoughts and feelings, or even her thoughts and feelings in times past. Rather, they were offered as proof of an act committed by someone else as evidence that she was dying of poison by her husband. The jury is incapable of distinguishing in its mind between these declarations as mere indications of a state of mind and as pointing the finger of guilt at Dr. Shepard. The ruling in Mutual Life Ins. Co. v. Hillman represents the high-water line beyond which courts may not go; in that case, the testimony looked forward to prove an occurrence. Here, it looks backward to a past act, and, more importantly, to an act by someone not the speaker.

Forfeiture of Wrongdoing United States v. Gray, 405 F.3d 227 (4th Cir.), cert. denied, 546 U.S. 912 (2005). Summary: Issue: Holding: Notes:

Rule 804(b)(6): Forfeiture by Wrongdoing Public Comments: The big picture: 801(d)(1) statements that are not hearsay prior statements of a testifying witness 801(d)(2) Statements that are not hearsay opposing Partys Statements (Party Opponent Exception) 803 exceptions where availability is immaterial (unrestricted hearsay exceptions; we dont care if the declarant is unavailable); 804 unavailable witness 807 residual (catch all) exception Hearsay Exceptions Under Rule 803: Availability of Declarant Immaterial Present Sense Impressions and Excited Utterances FRE 803(1) & (2) Present sense impression a statement describing or explaining an event or condition, made while or immediately after the declarant perceived it. o Purpose: to capture in court, the spontaneous, simultaneous description of what happened. o Theory not enough time to deliberate to develop a story with a conscious falsehood. o Declarant must have personal knowledge. Excited utterance a statement relating to a starling event or condition, made while the declarant was under the stress of excitement that it caused o See a lot of this is criminal cases. Note: both 803(1) and (2) have a content and time limit to them. o 803(1) Content limitation describe or explain the event/condition Time limit during or immediately after the declarant perceived the event or condition o 803(2) startling event or condition (not required for 803(1)) Content limitation relates to the startling event/condition Time limit made while the declarant was under the stress or excitement of the startling event /condition (the standard is the duration of the state of excitement) Problem 7.29 Statement 1 lunchtime statement That dog just bit me!

Hearsay out of court (over the phone) statement (That dog just bit me!) submitted to prove the thing asserted (that the dog owned by the Knollers bit Whipple) Admissible under 803(1)? o Maybe there is no indication that Whipple called Smith immediately after the incident; however, Whipple said just. That probably would not be enough to get it in unless there was some indication that Smith was called as soon as it happened. Admissible under 803(2)? o Yes o Content the utterance was made regarding the startling event/condition being bitten. o Time limit she was still agitated, so it was made while Whipple was under the stress or excitement of being bitten. Statement 2 Whipple told Smith she scolded Noel Not hearsay not being offered to prove the matter for which it is asserted Admissible under 803(1)? Admissible under 803(2)? Statement 3 You need to control your dog! Not hearsay not being offered to show that the dog mauled Whipple to death; offered to show that the Knollers were on notice. Statement 4 As I was walking by, the dog lunged at me. I put my hand out, and the dog bit me. Hearsay being offered to prove that the dog bit Whipple Admissible under 803(1)? No. o Content statement made the utterance described the event o Time limit it was not made during or immediately after the attack. Admissible under 803(2)? No o Content yes; relates to the startling event/condition o Time limit maybe; the standard is the duration of the stress/excitement of the event, so it would depend on whether she was still under the stress/excitement of the event. Several hours had passed, so it is doubtful that a judge would determine it fit within the time frame, however. Statement 5 she was lucky that she had escaped more serious injury Hearsay Statement 6 thank God I had my sports watch on Hearsay out of court (at home the night of the attack) statement (Thank God I had my sports watch on) being offered to prove the matter asserted (that Whipple had been bitten by the dog before, inferring viciousness of the attack) Admissible under 803(1)? No o Content statement described the event o Time limitation not made during or immediately after the attack Admissible under 803(2)? Probably not o

Problem 7.30 Problem 7.31 Statements of Then-Existing Conditions 803(3) FRE 803(3) a statement of the declarants then-existing state of mind (such as motive, intent, or plan) or emotional, sensory, or physical condition (such as mental feeling, pain, or bodily health), but not including a statement of memory or belief to prove the fact remembered or believed unless it relates to the validity or terms of the declarants will. Mutual Life Insurance Co. v. Hillman Summary: Evidence in dispute: Relevant? Being offered to prove: Holding: PROBLEM 7.33 Shepard v. United States Summary: Evidence in dispute: Relevant? Being offered to prove: Holding:

Mutual Life Insurance Co. v. Hillmon: Historical Postscript The Hillmon Doctrines Rationale Suspicions of Murder

Statements for Medical Diagnosis FRE 803(4) PROBLEM 7.34 (1) Lawyers testimony that Browning said he had fallen and hit his head No not made for medical diagnosis (2) Doctors testimony that Browning said he had fallen and hit his head - Yes FRE 803(A) it was made for and pertinent to medical diagnosis or treatment.

(3) Doctors testimony that Browning said Maples had pushed him - probably FRE 803(4)(B) a statement to describe the inception of the injury; also because it was a result of actions of the medical caregiver (custodian/one in a position of trust), one could argue that this was part of the medical treatment. However, the committee notes state that attribution of fault should be excluded. United States v. Iron Shell Summary: Evidence in dispute: statements Lucy made to Dr. Hopkins Relevant? yes Being offered to prove: that Iron Shell assaulted Lucy Hearsay? Yes Holding: Notes: Could also have used the hearsay exception of excited utterance, but did not raise the issue. PROBLEM 7.35 PROBLEM 7.36

Refreshing Memory and Recorded Recollections FRE 803(5) a record that (A) Is on a matter the witness once knew about but now cannot recall well enough to testify fully and accurately; (B) Was made or adopted by the witness when the matter was fresh in the witnesss memory; and (C) Accurately reflects the witnesss knowledge. If admitted, the record may be read into evidence but may be received as exhibit only if offered by an adverse party. See also FRE 612. PROBLEM 7.38 Sullivans testimony she wrote the license plate number on the envelope after Menandier called out. Relevant? Yes, if admitted, it tends to prove that the defendants car was the one that hit the pedestrian. Conditional relevance conditional on the numbers being the actual ones given by Menandier. Admissible under FRE 803(5)? Was it (A) on a matter the witness once knew about but now cannot recall well enough to testify fully and accurately? yes (B) made or adopted by the witness when the matter was fresh in the witnesss memory? yes

(C) accurately reflecting the witnesss knowledge? Maybe; this may be where the conditional relevance becomes important. a. She acknowledges that she wrote the numbers, but cannot recall the numbers. This would be conditional on whether Menandiers testimony corroborated the memorializing of the number. b. She says she remembers clearly that Menandier called out the number and she wrote down the numbers that he called out. Committee Notes: the rule should also be interpreted to cover other situations involving multiple participants, e.g., employer dictating to secretary, secretary making memorandum at direction of employer, or information being passed along a chain of persons. The record may be read into evidence. It cannot be admitted as an exhibit because it was not offered by an adverse party.

Menandiers testimony he memorized it by repeating it; remembered it after glancing at the envelope. (A) Is on a matter the witness once knew about but now cannot recall well enough to testify fully and accurately; yes (B) Was made or adopted by the witness when the matter was fresh in the witnesss memory? yes (C) Accurately reflects the witnesss knowledge. Yes, as demonstrated by his reciting it again. Relevant? Yes Elements satisfied? Yes Brancatellas options: o Get a copy of the envelope make sure that there was no coaching o Admit into evidence o Remind jury that Menandier could not remember. He saw the numbers and then recited them. Could the prosecution admit it into evidence after Menandiers testimony? Still no; only the adverse party could get it admitted.

Johnson v. State Summary: Arnold Johnson was convicted of the capital murder of Frank Johnson. Reginald Taylor was a witness and gave a statement to the police immediately after the murder. At trial, Taylor was called to testify, but was uncooperative and could not remember anything. Evidence in dispute: the signed statement given to police after the murder. Relevant? Yes tends to prove that Arnold committed the murder. Being offered to prove: that Arnold committed the murder. Hearsay? Out of court (at the police station) statement (the signed statement detailing how Arnold killed Frank) submitted to prove the thing asserted (that Arnold murdered Frank).

Rule: FRE 803(5) Reasoning: Four elements to meet Rule 803(5): The witness must have had firsthand knowledge of the event The written statement must be a memorandum made at or near the time of the event while the witness had a clear and accurate memory of it, The witness must lack a present recollection of the event, and The witness must vouch for the accuracy of the written memorandum. Holding: The last element wasnt met, so the statement should not have been admitted into evidence. It was inadmissible hearsay. Reversed and remanded. Notes: Could have used 804(a)(3) unavailable by not remembering 804(b)(6) forfeiture by wrongdoing Excited utterance depending on the time frame; counter made at the police station surrounded by uniformed officers; he would feel more protected and should have calmed down. Business Records FRE 803(6) Palmer v. Hoffman: Historical Backdrop Business records exception purpose to avoid the difficulty of finding witnesses who could speak from firsthand knowledge about a businesss routine and often nondescript activities. Justification for the business records exception unusual trustworthiness. They connate o A regularity serving to counteract the possible temptation to misstatements; o A situation which would lead to detection of falsification, so that misstatements cannot safely be made; o A relationship, when a writing is made by an employee under a duty to his employer, which includes the risk of censure and disgrace for misstatements. An essential condition of the admission of such hearsay is the circumstantial guaranty of trustworthiness consisting of the accuracy ensured by the nature of the regular system of entries. Palmer v. Hoffman Summary

Howard Hoffman and his wife Inez were on their way home to Hartford. Hoffman crossed the railroad tracks at the Elkey-Buckley railroad crossing when he was hit by a train, injuring him and killing his wife. Hoffman sued the railroad; the railroad said its agents acted with due care and asserted contributory negligence. the interview given by the engineer (conductor) employed by the railroad.

Issue Evidence in dispute

The engineer died before trial. It was offered as part of the business records exception because it was customary to take the statement of employees after an accident. (the statement was taken 2 days after the accident, given to the Assistant Superintendent of the Railroad) Relevant? yes Being offered to prove: that the railroad was not negligent Hearsay? Yes out of court statement submitted to prove the matter alleged. Being used to prove Hearsay? Exception Holding Reasoning

the report was not for the systematic conduct of the enterprise as a railroad business and was properly excluded. The record was not made for the systematic conduct of the business of the business. An accident report is not typical of entries made systematically or as a matter of routine to record events or occurrences, to reflect transactions with others, or to provide internal controls. The fact that a company makes a business out of recording its employees versions of their accidents does not put those statements in the class of records made in the regular course of the business within the meaning of the Act. o Regular course of business regular course of conduct Legislative history o 45 U.S. C. 40 gives the Interstate Commerce Commission authority to investigate and make reports of accidents. It provided that inclusion of such accident reports were not to be admitted as evidence or used for any purpose in any suit or action for damages growing out of any matter mentioned in said report of investigation. Could it have come in under FRE 804(a)(4)? The declarant was deceased.

My notes, thoughts, questions PROBLEM 7.39 Summary:

Evidence in dispute:

Being used to prove? Hearsay?

Donohue bought a gas-powered mower at Garden Depot. Later she sued the mowers manufacturer for injuries she sustained when the pull cord came loose while she was trying to start the mower. Donohue claimed the sudden release of the cord caused her to fall over backward and strike her head. 1) Two merchandise-return records from the service counter at Garden Depot. 2) Merchandise-return ticket completed by Donahue stating the reason for the return was because the grass chute often clogs up. 1) That this make and model mowers had the same defect as the one she bought. 1) Yes 2) Yes

Exception? Analysis

1) Maybe 2) Statement against interest 1) Clerk wrote in form, Customer said the cord was defective. This is hearsay within hearsay. a. The form itself meets the requirement of an 803(6) business record exception. i. made at or near the time by or from information transmitted by someone with knowledge. ii. The record was kept in the course of a regularly conducted activity of a business, organization, occupation, or calling, whether or not for profit; iii. Making the record was a regular practice of that activity; iv. All of the conditions were shown by the testimony of a person qualified witness, the manager. b. The statement on the form probably does not fall within the exception. i. Did not verify the truthfulness/trustworthiness of the statement (not based on personal knowledge?). 1) Will probably not be admitted because the statement on the form does not appear to fall within an exception. Donohue could try to get it in under FRE 807 (The Residual Exception), but that would be very difficult and probably would not work.

Conclusion

United States v. Vigneau Summary: Evidence in dispute Vigneau was convicted of conspiracy to distribute marijuana and 21 counts of money laundering on specific occasions and conspiracy to launder money. 21 Western Union To Send Money forms primarily used in support of the money laundering charges. The forms included Vigneaus name, address and phone number. Those forms specifically corresponded to the 21 specific counts of money laundering on which Vigneau was convicted. Yes would tend to make the allegation that Vigneau was involved in money laundering more probable. Being used to prove: that he wired money Yes

Relevant? Being used to prove? Hearsay? Exception? Rule: Holding: Reasoning:

the forms should not have been admitted for their truth because there were no safeguards

Public Records and Reports FRE 803(8) and (10) Beech Aircraft Corp. v. Rainey, 488 U.S. (1988). Summary a Navy training aircraft crashed during touch-and-go training exercises, killing the flight instructor and student. The pilots surviving spouses brought a products liability suit against Beech Aircraft Corp, the planes manufacturer, and Beech Aerospace Services, which serviced the plane under contract with the Navy. whether FRE 803(8)(C), which excepts investigatory reports from the hearsay rule, encompasses evaluative conclusions or opinions. statements in an investigative report prepared by Lieutenant Commander William Morgan that indicated that the fatal crash was a result of pilot error. That the crash was a result of pilot error. Yes FRE 803(8)(C) Public records the statements within the report are admissible; FRE 803(8)(C) exception includes evaluative conclusions or opinions. The statements should have been admitted. Must start with statutory interpretation. o Not apparent that the term factual findings should be read to mean simply facts. The language of the rule does not compel rejection of the interpretation that factual findings includes conclusions or opinions that flow from a factual investigation. o The language of the rule does not state that factual findings are admissible, but that reports setting forth factual finds are admissible. Legislative history not helpful o House Judiciary Committee stated that the phrase factual findings be strictly construed and that evaluations or opinions contained in public reports shall not be admissible under this Rule. o The Senate Committee said the opposite. Advisory Committees comments do not mention any dichotomy between statements of fact and opinions or conclusions. o Nowhere in its comments is there the slightest indication that it even considered the solution of admitting only factual statements from such reports. Rather, the Committee referred throughout to reports, without any such differentiation regarding the statements they contained. o Final paragraph of the report on this Rule:

Issue

Evidence in dispute

Being used to prove Hearsay? Exception Holding

Reasoning

the rule assumes admissibility in the first instance It provides ample provisions for escape if sufficient negative factors are present. Limitations and safeguards: o The FRE requirement that reports contain factual findings bars the admission of statements not based on factual investigation. o The trustworthiness provision requires the court to make a determination as to whether the report, or any portion thereof, is sufficiently trustworthy to be admitted. Safeguards are built into other provisions of the FRE. o Relevance and prejudice o Ultimate safeguard the opponents right to present evidence tending to contradict or diminish the weight of the conclusions. The FRE favors admissibility. Allowing the report is in line with the broad approach of the FRE.

Police Reports and Business Records FRE 803(8)(B) and 803(8)(C) do not extend to the reports of law enforcement personnel or evaluative reports if offered against the defendant in a criminal case. United States v. Oates, 560 F.2d (2nd Cir. 1977). Summary: drug case Evidence in dispute government chemists report Being used to prove? The material in question was drugs Hearsay? yes Exception? Business records exception? Rule: Law enforcement reports are absolutely inadmissible against defendants in criminal cases. Holding: the chemists report and worksheet do not satisfy the standards of FRE 803(8) or FRE 803(6) Reasoning:

United States v. Hayes, 861 F.2d 1225 (10th Cir. 1988). Summary: Evidence in dispute: Hearsay? Exception?

Holding: Reasoning:

United States v. Weiland, 420 F.3d 1062 (9th Cir. 2005), cert. denied, 547 U.S. 1114 (2006). Summary: Evidence in dispute: Hearsay? Exception? Holding: Reasoning: RESIDUAL EXCEPTION FRE 807 FRE 807 Dallas County v. Commercial Union Assurance Co., 286 F.2d 388 (5th Cir. 1961). Summary: Evidence in dispute: newspaper article describing a fire at the courthouse. Dallas County claimed the damage was a result of a fire from a lightning strike. The insurance company claims that the damage was a result of an earlier fire that occurred long before they insured the building. Hearsay? Exception? Holding: Reasoning:

Afterthoughts: CONFRONTATION AND COMPULSORY PROCESS The Confrontation Clause and Hearsay The sixth amendment provides that in all criminal prosecutions, the accused shall enjoy the right to be confronted with the witnesses against him Crawford v. Washington, 541 U.S. 36 (2004) Summary: Michael Crawford was convicted of stabbing Kenneth Lee, the man who allegedly tried to rape his wife, Sylvia. Michael was charged with assault and attempted murder. He claimed self-defense. At trial, the prosecution presented a tape-recorded statement of Michaels wife even though she did not testify (spousal privilege). The jury convicted Michael of assault. Issue: Whether the States use of Sylvias statements violated the Confrontation Clause. Evidence in dispute: the tape-recorded statement Sylvia gave to police. o Relevant? Yes, it tended to make the charge of assault more probable and cast doubt on the claim of self-defense.

Hearsay? Yes, it was an out of court (at the police station) statement (tape recording of Sylvias recollection of the event) being offered to prove that which it asserts. o Exception? o Confrontation clause issue? Yes Holding: The admission of Sylvias statement was a violation of Michaels sixth amendment right to cross-examine witnesses. Reasoning: History shows: o The principal evil at which the Confrontation Clause was directed was the civil-law mode of criminal procedure, and particularly its use of ex parte examinations as evidence against the accused. Not all hearsay implicates the Sixth Amendments core concerns. Off-hand comments might be unreliable and thus a good candidate for exclusion under the hearsay rules, but bears little resemblance to the civil-law abuses the Confrontation Clause targeted. Ex parte examinations might sometimes be admissible under modern hearsay rules, but the Framers certainly would not have condoned them Confrontation clause applies to witnesses against the accused. Witnesses are those who bear testimony Testimony is typically a solemn declaration or affirmation made for the purpose of establishing or proving some fact. Even if the Sixth Amendment is not solely concerned with testimonial hearsay, that is its primary object, and interrogations by law enforcement officers fall squarely within that class. The constitutional text, like the history underlying the common-law right of confrontation, thus reflects an especially acute concern with a specific type of out-of-court statement. o The Framers would not have allowed admission of testimonial statements of a witness who did not appear at trial unless he was unavailable to testify, and the defendant had had a prior opportunity for cross-examination. The text of the Constitution does not suggest any open-ended exceptions from the confrontation requirement to be developed by the courts. Most of the hearsay exceptions covered statements that by their nature were not testimonial for example, business records or statements in furtherance of a conspiracy. The Court does not infer that the Framers thought exceptions would apply even to prior testimony. Case law o Mattox v. United States prior trial testimony or preliminary hearing testimony is admissible only if the defendant had an adequate opportunity to cross-examine. o California v. Green reaffirmed Mattox

Pointer v. Texas reaffirmed Mattox Bruton v. United States testimony was excluded where the government had not established the unavailability of a witness. o Ohio v. Roberts admitted testimony from a preliminary hearing at which the defendant had examined the witness o Lilly v. Virginia excluded testimonial statements that the defendant had no opportunity to test by cross-examination o Bourjaily v. United States admitted statements made unwittingly to an FBI informant after applying a more general test that did not make prior cross-examination an indispensable requirement Rationales o

o o

Davis v. Washington Summary (The Supreme Court combined these cases because they were so closely related.) Facts Michelle McCottry made a domestic disturbance call to 911. In the call, she described an assault of her former boyfriend, Adrian Davis and asked for help. The operator got the vital information and then, after McCottry told the operator Davis had left, told her to stop talking and answer my questions. She then told McCottry that police were on their way.

Hammon v. Indiana Facts Amy Hammon called 911 regarding a domestic disturbance. When the police arrived, the situation had calmed down. The police separated Amy and Herman and questioned them separately. Herman tried to interject into Amys questioning, but was not allowed. After Amys questioning, the police had her complete a battery affidavit and sign it. She did not appear in court. The affidavit was submitted as evidence against Herman.

Davis v. Washington Procedural History Davis was charged with felony violation of a domestic no-contact order. At trial, the States only witnesses were the two police officers who responded to the 911 call. They testified that McCottry exhibited injuries that appeared to be recent, but neither officer could testify as to the cause of the injuries. McCottry presumably could have testified, but she did not appear. The court admitted the 911 tape over Davis objection.

Evidence in dispute Relevant? Hearsay?

The tape of the 911 call.

Yes It tends to make the allegation of domestic abuse more likely. Yes it was an out-of-court (by

Hammon v. Indiana Procedural History The State charged Hershell with domestic battery and with violating his probation. The trial court admitted the battery affidavit that Amy filled out and signed after telling officers about the incident. o Affidavit present sense impression o Statements excited utterances The trial judge found Hershel guilty on both charges. The Indiana Court of Appeals affirmed in relevant part. The Indiana Supreme Court also affirmed, concluding o Amys statement was admissible for state-law purposes as an excited utterance; o that a testimonial statement is one given or taken in significant part for purposes of preserving it for potential future use in legal proceedings, where the motivations of the questioner and declarant are the central concerns; and o that Amys oral statement was not testimonial under these standards. o The affidavit was testimonial and wrongly admitted, but the error was harmless beyond a reasonable doubt, largely because the trial was to the bench. The battery affidavit completed by Amy made during questioning in her home at her dining room table. Yes It tends to make the allegation of domestic abuse more likely. Yes - it was an out-of-court (Amy

Exception? Issue

Rule

Problem for Court

Holding

Reasoning

Davis v. Washington telephone) statement (conversation memorialized in the transcript) offered to prove the truth of the matter asserted (that Davis physically assaulted her). 803(1) present sense impression Whether the admission of the 911 tapes were a violation of Davis 6th Amendment right to confront witnesses against him. 6th Amendment in all criminal prosecutions, the accused shall enjoy the right to be confronted with the witnesses against him. What are testimonial statements? Statements are nontestimonial when made in the course of police interrogation under circumstances objectively indicating that the primary purpose of the interrogation is to enable police assistance to meet an ongoing emergency. There was no 6th Amendment Confrontation Clause Issue. The statements were non-testimonial. judgment affirmed The first part of the conversation presented no 6th Amendment issue The second part, after the operator told her to stop talking and answer questions, may have been testimony and maybe ought to have been redacted. The Court mentioned it but didnt decide. The 6th Amendment applies to testimonial statements by a witness submitted against a criminal defendant. The primary purpose test established that the statements were not testimonial. o It was a cry for help, not an explanation of the event. The statement was not formal as opposed to Crawford.

Hammon v. Indiana Hammons living room table) statement (battery affidavit) offered to prove the truth of the matter asserted (that Hershel physically assaulted her). 803(2) excited utterance Whether the admission of the statement was a violation of Hammons 6th Amendment right to confront witnesses against him.

Statements are testimonial when the circumstances objectively indicate that there is no such ongoing emergency, and that the primary purpose of the interrogation is to establish or prove past events potentially relevant to later criminal prosecution.

Yes, the affidavit was testimonial. The admission of the affidavit was a violation of the 6th Amendment Confrontation Clause Issue reversed and remanded

The statements were not much different than those in Crawford. Objectively viewed, the primary (sole?) purpose of the interrogation was to investigate a possible crime, not to assess an emergency situation. o There was no emergency in progress; the interrogating officer testified that he heard no arguments or

Dissent (Thomas)

Davis v. Washington o Made during the emergency was speaking about events as they were occurring o Was facing an ongoing emergency it was a call for help against bona fide physical threat o The statements were necessary to be able to resolve the present emergency, rather than simply learn what had happened o The level of formality frantic answers provided over the phone, in an environment that was not tranquil or even safe (as opposed to Crawford The Courts standard is not only disconnected from history and unnecessary to prevent abuse; it also yields no predictable results to police officers and prosecutors attempting to comply with the law.

Hammon v. Indiana crashing and saw no one throw or break anything. o When officers arrived, Amy told them things were fine, and there was no immediate threat to her person. o When the officer questioned Amy, it was to determine what had happened not what was happening The statements were a substitute for live testimony because they did precisely what a witness does on direct examination.

The Courts determination that the evidence against Hammon must be excluded extends the Confrontation Clause beyond the abuses it was intended to prevent. The inconsistent results between Davis and Hammon illustrate the difficulty of applying the Courts primary purpose*s+ of the investigation.

Notes/Thoughts Primary-Purpose Test: Statements are nontestimonial when made in the course of police interrogation under circumstances objectively indicating that the primary purpose of the interrogation is to enable police assistance to meet an ongoing emergency. Statements are testimonial when the circumstances objectively indicate that there is no such ongoing emergency, and that the primary purpose of the interrogation is to establish or prove past events potentially relevant to later criminal prosecution. Whose Purpose Controls? The Court rejected the notion of defining the testimonial nature of a statement according to the intent of the declarant. Asking whether the declarant spoke in contemplation of legal proceedings would entangle the courts in a multitude of difficulties. In stating the primary purpose test what circumstances objectively indicate.

Look at what the officers are doing, but the declarants purpose must bear on the testimonial nature of her report.

Problem 8.1 Evidence in dispute: a letter Julie Jensen wrote and gave to her neighbor in the event anything happened to her. Relevant? Yes Hearsay? Yes Exception? Unavailable witness 6th Amendment violation? Primary purpose to implicate her husband Testimonial? Yes it was to help the police investigate and solve a crime, unless you subscribe to the view that it is only statements produced by interrogations can be testimonial. In that case, argue that there was no interrogation, so the statement was not testimonial. This generally does not hold up. Generally, statements like this are considered testimonial using the reasonable declarant test. Problem 8.3 Best arguments could not cross-examine the witness the statement was testimonial o the officer took her to the hospital and remained there o the waiver specifically stated the exam and collections would be used as evidence at trial. o Looks like evidence gathered to establish or prove past events relevant to a later prosecution. Prosecution argument The officer was not allowed in to ask questions The primary purpose for the information was for the treatment of the rape o It was a nurse getting the information o She was in a hospital, not a police station Hearsay exception medical diagnosis (Unavailable witness would not work because in order to overcome the unavailability, the defendant had to have had the opportunity to cross examine the witness.) The Confrontation Frontier: Recent and Remaining Battles in the Crawford Revolution Regarding statements of child victims regarding abuse, most courts have adopted a Davis-style focus on the questioners purpose and pay little or no heed to the childs intent. And when the questioner is a parent or doctor or nurse, most courts have deemed the childs responses to be nontestimonial. Certificates and Affidavits Crawford business records are by their nature . . . not testimonial.

Post-Crawford courts have rejected Confrontation Clause objections to classic business records. Yet some business records seem to fit any definition of testimonial statements. o Drug analysis, blood analysis, motor vehicle registrars, tax examinations, etc. are usually completed in the regular course of business but also to uncover evidence for prosecutorial use and documented in formal reports. Lower courts are split almost evenly on the testimonial nature of formal reports o Testimonial People v. Rogers the test was initiated by the prosecution and generated by the desire to discover evidence against defendant. o Non-testimonial State v. Dedman although the report is prepared for trial, the process is routine, non-adversarial, and made to ensure an accurate measurement. Many post-Davis courts focus on the purpose of the documents creation, sharpening the debate on whether formal reports are testimonial or not. o Reports intended to substitute for trial testimony by narrating past events potentially relevant to later criminal prosecution tend to be deemed testimonial, especially when written at police departments or prosecutors request bearing on a specific suspect. o Certificates of state crime lab chemists reporting the results of drug or blood tests rarely have won admission as nontestimonial. Most post-Davis courts condemn such reports as testimonial because their primary purpose is to prove an element of the prosecutions case at trial and to fill a witnesss role in doing so.

Whorton v. Bockting (2007) the justices ruled unanimously that Crawford has no retroactive force in cases made final after direct appellate review. Analyzed it under Teague v. Lane: Newness Crawford is not merely new, it is flatly inconsistent with the prior governing precedent, Roberts, which Crawford overruled. Accuracy the court acknowledged that Crawford might avoid some of the inaccurate trial outcomes by excluding unreliable testimonial statements that slipped past trial judges under Roberts, but also admits that nontestimonial statements that are unreliable would be let in, thus lessening the accuracy of trial judgments. Giles v. California (2008) Summary: Dwayne Giles was on trial for killing his former girlfriend. The lower courts allowed in statements which she made to a police officer who responded to a domestic-violence complaint about three weeks before her death, and which was now offered as evidence against him. The lower courts held that in killing his former girlfriend, Dwayne Giles forfeited his right to cross-examine her about a claim that he had beaten her. Procedural history - California Supreme Court The prosecution did not need to show his purpose in killing her.

Equity one should not profit from ones wrongs. This applies whether or not those wrongs aimed to eliminate a trial witness.

Holding: - the constitutional doctrine of forfeiture by wrongdoing, like Rule 804(b)(6), requires a showing that the defendant acted with the purpose of preventing the declarant from testifying. Concurring opinions: Thomas agreed with the statement regarding forfeiture, but emphasized a much narrower understanding of the category of testimonial statements than the one embraced by the Court in Crawford: o The Confrontation Clause applies only to such formalized hearsay as past testimony and custodial, Mirandized police interrogations. Alito wrote more tentatively to make clear that, like Justice Thomas, he was not convinced that the out-of-court statement at issue fell within the Confrontation Clause in the first place. Souter and Ginsburg endorsed the Courts interpretation of the forfeiture doctrine, but explained his position in terms of modern procedure. o Statements are admissible solely because the defendant kept the witness out of court by committing homicide. o Admissibility of the victims statement to prove guilt would turn on finding the defendant guilty of the homicidal act causing the absence. o Evidence that the defendant killed would come in because the defendant probably killed. o Conclusion equity demands something more than question begging. Liberality in Domestic-Violence Cases Common thread among all the concurrences the justices seem prepared to infer from the history of abuse the defendants purpose to keep his victim from the stand. Michigan v. Bryant, 131 S.Ct. 1143 (2011) Evidence in dispute: statements made by gunshot victim Anthony Covington to police that Richard Bryant shot him. Hearsay? Yes; out of court statement submitted for the truth of what it asserts. The statement (Rick shot him and where) was made in the floor of a gas station (out of court) and was submitted to prove that which it asserts (that Rick was the shooter). Exception? Dying declaration Covington died Confrontation clause violation? Primary purpose of the questioning Police said it was to assess the situation. They needed to know if the shooter was in the area and if people were in danger. Declarant to convey information for treatment

Defendant said it was to gather evidence. The police knew the gravity of the situation, and there was no ongoing emergency.

Holding:

Firm Footholds and the Confrontation Flowchart (RB 604 607) Statements Not Offered for their Truth: o not a Confrontation Clause issue o Crawford the Clause does not bar the use of testimonial statements for purposes other than establishing the truth of the matter asserted. Statements Offered in Civil Cases or against the Prosecution o Not a Confrontation issue o The 6th Amendment only protects only criminal defendants Past statements of Trial Witnesses o Crawford when the declarant appears for cross-examination at trial, the Confrontation Clause places no constraints at all on the use of his prior testimonial statements. Statements by Unavailable Declarants Subject in the Past to Cross-Examination by the Defendant o A defendants past chance to cross-examine a declarant who proves unavailable at trial satisfies the Confrontation Clause Statements Admitted by Forfeiture o Giles v. California a defendant who intended by his wrongdoing to keep the declarant from testifying at trial forfeits his confrontation right. Nontestimonial Statements o Formalized Testimonial Materials the Confrontation Clause excludes formalized testimonial materials, such as affidavits, depositions, prior testimony, or confessions. o Casual and Offhand Remarks and Coconspirators Crawford deemed nontestimonial casual remarks to an acquaintance, off-hand, overheard remarks, and statements in furtherance of a conspiracy. The purpose of the statement/remark was not to provide testimony. Interrogation: Primary Purpose Test from Davis for testimonial statements combined perspective (declarant and officer; from the objective person) Formality (Thomas) Medical condition Scope of danger/presence of gun? (i.e., is it during an on-going emergency?) Bryant - Reliability/hearsay law -> maybe no Confrontation Clause implication Forensic Analyst Reports in Melendez-Diaz and Bullcoming

Non-interrogation: Purpose of the statement? o Majority of the court says, if testimonial -> implicates Confrontation Clause o Casual remark are non-testimonial -> no Confrontation Clause Conventional Eyewitness? -> may implicate Confrontation Clause

You might also like